Saunders-ATI-Davis-Intrapartum Study Guide

Ace your homework & exams now with Quizwiz!

During a vaginal exam, the nurse palpates fetal buttocks that are facing the left posterior and are 1 cm above the ischial spines. Which of the following is consistent with this assessment? 1. LOA -1 station 2. LSP -1 station 3. LMP +1 station 4. LSA +1 station

(D-10) 2. is correct. - LSP -1 station. Note: The fetal buttocks (S or sacrum) are facing toward the mom's left posterior (LP) and the buttocks are at -1 station or 1 cm above the ischial spines.

The labor room nurse assists with the administration of a lumbar epidural block. How should the nurse check for the major side effect associated with this type of regional anesthesia?

**Monitoring the mother's blood pressure **Rationale: A major side effect of regional anesthesia is hypotension, which results from vasodilation in the lower body and a reduction in venous return. After regional anesthesia, the blood pressure is taken every 1 to 2 minutes for 15 minutes and then every 10 to 15 minutes. (S-43)

The nurse has a routine prescription to administer an injection of phytonadione (Vitamin K) to the newborn. Which statement made by the new mother indicates that teaching on this medication was effective?

--"I know that this medication is used to prevent clotting abnormalities in the newborn." Rationale: Vitamin K is given to the newborn to prevent clotting abnormalities. Vitamin K is usually produced by bacteria in the gastrointestinal tract, which is sterile in the newborn. (Saunders Maternal Meds)

The nurse gave an intramuscular dose of methylergonovine to a client following delivery of an infant. The nurse determines that this medication had the intended effect if which finding is noted?

--Improved uterine tone Rationale: Methylergonovine is an ergot alkaloid that is given following delivery to treat postpartum hemorrhage. It acts by vasoconstricting arterioles and directly stimulating uterine muscle contractions. Blood pressure may increase, but this is not the intended therapeutic effect. (Saunders Maternal Meds)

___ = Paced breathing technique during which a woman breathes at approximately 32-40 breaths per minute interspersed with blowing out of air in a ratio of 3:1 or 4:1; this technique enhances concentration.

Birthing from within

Proteinuria with SEVERE preeclampsia

r/t renal vasospasms - narrowing of glomerular capillaries which leads to decreased renal perfusion and decreased GFR (glomerular filtration rate) due to damage to glomeruli. This results in proteinuria where protein leaks across the plasma membrane. The tubules are unable to reabsorb it. **The degree of PROTEINURIA reflects the severity of the disease. Spilling 1+ of protein is significant to begin treatment. **Oliguria and tubular necrosis my precipitate acute renal failure. **Nurse should monitor urine output is at least 30ml / hour.

Positions affect the woman's anatomic and physiologic adaptations to labor. Frequent changes in position ___.

relieve fatigue, increase comfort, and improve circulation.

3. Continuation of Pitty Pitocin story: Pitty gets so nervous with all that is happening, her blood pressure increasing, her fluid retention, that she experiences _____ (Lecture handout)

tetantic contractions. She goes into CARDIAC ARRHYTHMIAS causing Pitty's baby to experience OXYGEN hunger and this results in FETAL HEART IRREGULARITIES.

One hour ago, a multipara was examined with the following results: 8 cm, 50% effaced, and+1 station. She is now pushing with contractions and the fetal head is seen at the vaginal introitus. The nurse concludes that the client is now 1. 9 cm dilated, 70% effaced, and+2 station. 2. 9 cm dilated, 80% effaced, and +3 station. 3. 10 cm dilated, 90% effaced, and+4 station. 4. 10 cm dilated, 100% effaced, and+5 station

(D-14) 10 cm dilated, 100% effaced, and+5 station RATIONALE: The cervix is fully dilated and fully effaced and the baby is low enough to be seen through the vaginal introitus.

The nurse is caring for a nulliparous client who attended Lamaze childbirth education classes. Which of the following techniques should the nurse include in her plan of care? Select all that apply. 1. Hypnotic suggestion 2. Rhythmic chanting 3. Muscle relaxation 4. Pelvic rocking 5. Abdominal massage

(D-15) 3, 4, and 5 are correct. 3. Muscle relaxation 4. Pelvic rocking 5. Abdominal massage RATIONALE:

Which of the following responses is the primary rationale for the inclusion of the information taught in childbirth education classes? 1. Mothers who are performing breathing exercises during labor refrain from yelling. 2. Breathing and relaxation exercises are less exhausting than crying or moaning. 3. Knowledge learned at childbirth education classes helps to break the fear-tension-pain cycle. 4. Childbirth education classes help to promote positive maternal-newborn bonding.

(D-16) 3. Knowledge learned at childbirth education classes helps to break the fear-tension-pain cycle RATIONALE:

The childbirth educator is teaching a class of pregnant couples the breathing technique that is most appropriate during the 2nd stage of labor. Which of the following techniques did the nurse teach the women to do? 1. Alternately pant and blow 2. Take rhythmic, shallow breaths 3. Push down with an open glottis 4. Do slow chest breathing

(D-17) 3. Push down with an open glottis RATIONALE:

A woman who states she "thinks" she is in labor enters the labor suite. Which of the following assessments will provide the nurse with the most valuable information regarding the client's labor status? 1. Leopold's maneuver 2. Fundal contractility 3. Fetal heart assessment 4. Vaginal exam

(D-2) 4. Vaginal exam 1. Leopold's maneuver -assesses fetal position not labor progress 2. Fundal contractility - assesses uterine contractions but is not the most valuable information regarding labor status. 3. Fetal heart assessment - is critically important, but assesses fetal well-being 4. Vaginal exam - provides the "most" valuable data regarding the progress of labor. This exam can determine whether the woman is in true labor where cervical change occurs: effacement and dilation.

A client in labor, G2 P1001, was admitted 1 hour ago at 2 cm dilated and 50% effaced. She was talkative and excited at the time. During the past 10 minutes she has become serious, closing her eyes and breathing rapidly with each contraction. Which of the following is an accurate nursing assessment of the situation? 1. The client had poor childbirth education prior to labor. 2. The client is exhibiting an expected behavior for labor. 3. The client is becoming hypoxic and hypercapnic. 4. The client needs her alpha-fetoprotein levels checked.

(D-3) 2. The client is exhibiting an expected behavior for labor. *Test-taking tip: be familiar with different phases of the first stage of labor: latent, active, transition and typical client response in each phase.

A client has just arrived at the labor and delivery suite. Which of the following assessments must the nurse perform before contacting the HCP with the client's status? Select all that apply. 1. FHR 2. Contraction pattern 3. VS 4. Biophysical profile

(D-4) 1, 2, and 3 are correct. **The nurse should assess the fetal heart rate, the contraction pattern, and maternal vital signs before reporting the client's status to the HCP.

While performing Leopold's maneuver on a woman in labor, the nurse palpates a hard round mass in the fundal area, a flat surface on the left side, small objects on the right side, and a soft round mass just above the symphysis. Which of the following is a reasonable conclusion by the nurse? 1. The fetal position is transverse. 2. The fetal presentation is vertex. 3. The fetal lie is vertical. 4. The fetal attitude is flexed.

(D-5) 3 is correct - 3. The fetal lie is vertical. NOTE: Many OB assessments have a component that is sensual, and a component that is an interpretation or concept, such as Leopold's maneuver. The nurse must palpate and interpret what is felt into a concept.

When during the latent phase of labor should the nurse assess the FHR pattern of a low risk woman, G1 P0000? Select all that apply. 1. After vaginal exam. 2. Before administration of analgesics. 3. Periodically at the end of a contraction. 4. Every 10 minutes. 5. Before ambulating.

(D-6) 1, 2, 3, and 5 are correct. Except for invasive procedures, assessment of FHR pattern is the only way to evaluate the well-being of a fetus during labor. The FHR pattern should be assessed whenever there is a potential for injury to the baby or umbilical cord. In each of the correct answers there is a potential for cord compression or fetal injury.

The labor and delivery nurse performs Leopold's maneuvers. A soft round mass is felt in the fundal region. A flat object is noted on the left and small objects are noted on the right side of the uterus. A hard round mass is palpated above the symphysis. Which of the following positions is consistent with these findings? 1. LOA - Left occipital anterior 2. LSP - Left sacral posterior 3. RMA - Right mentum anterior 4. RSP - Right sacral posterior

(D-8) 1 is correct - LOA - Left occipital anterior 1. In the LOA position the fetal back is on the mom's left side, the small parts are on her right side, the buttocks are in fundal region, and the head is felt above her symphysis. 2. In LSP position the fetus buttocks (S-sacrum) are facing toward the mom's left posterior(LP), a hard round mass is felt in the fundal region, and a soft mass is felt above the symphysis. 3. In RMA position the fetus's face (or Mentum) is facing the mom's RA or right anterior, the small objects are felt on the right, with the flat area felt on mom's left side. 4. IN RSP position the fetus's sacrum (S or buttocks) is facing the mom's right posterior (RP) and a hard round mass is felt in the fundal region while the soft round mass is felt above the symphysis.

A nurse is caring for a laboring woman who is in transition. Which of the following S/S would indicate that the woman is progressing into the second stage of labor? Select all that apply. 1. Bulging perineum 2. Increased bloody show 3. Spontaneous rupture of membranes 4. Uncontrolled urge to push 5. Inability to breath through contractions.

(D-9) 1, 2, and 4 are correct. Notes: -The bulging perineum and the fully dilated cervix cause mom to feel strong urge to push -The amniotic sac can rupture at any time -The gravida's ability to work with her labor is more dependent on her level of pain and her preparation for labor than on stages / phases of labor.

A client enters the labor and delivery suite stating that she thinks she is in labor. Which of the following information about the woman should the nurse note from the woman's prenatal record before proceeding with the physical assessment? Select all that apply. 1. Weight gain 2. Ethnicity and religion 3. Age 4. Type of Insurance 5. Gravidity and parity

(Davis) 1, 2, 3, 5 are correct. 1. Weight gain changes can influence current status needs. 2. The client's ethnicity and religion b/c this allows the nurse to proceed in a culturally sensitive manner. 3. An older or younger client have specific needs related to their age that can influence labor and delivery. 5. Gravidity and parity records show client past history and any difficulties associated.

The nurse is monitoring a client who is in the active stage of labor. The nurse documents that the client is experiencing *labor dystocia*. The nurse determines that which risk factors in the client's history placed her at risk for this complication? Select all that apply. 1. Age 54 2. Body mass index of 28 3. Previous difficulty with fertility 4. Administration of oxytocin for induction 5. Potassium level of 3.6 mEq/L (3.6 mmol/L)

**1. Age 54, and 2. Body mass index of 28 **Rationale: Risk factors that increase a woman's risk for dysfunctional labor include the following: advanced maternal age, being overweight, electrolyte imbalances, previous difficulty with fertility, uterine overstimulation with oxytocin, short stature, prior version, masculine characteristics, uterine abnormalities, malpresentations and position of the fetus, cephalopelvic disproportion, maternal fatigue, dehydration, fear, administration of an analgesic early in labor, and use of epidural analgesia. -- Age 54 is considered advanced maternal age and a body mass index of 28 is considered overweight. -- Previous difficulty with infertility is another risk factor for labor dystocia. A potassium level of 3.6 mEq/L (3.6 mmol/L) is normal and administration of oxytocin alone is not a risk factor; risk exists only if uterine hyperstimulation occurs. (S-18)

The nurse in a labor room is assisting with the vaginal delivery of a newborn infant. The nurse should monitor the client closely for the risk of uterine rupture if which occurred? 1. Forceps delivery 2. Schultz presentation 3. Hypotonic contractions 4. Weak bearing-down efforts

**1. Forceps delivery **Rationale: Excessive fundal pressure, forceps delivery, violent bearing-down efforts, tumultuous labor, and shoulder dystocia can place a client at risk for traumatic uterine rupture. -- Schultz presentation is the expulsion of the placenta with the fetal side presenting first and is not associated with uterine rupture. -- Hypotonic contractions and weak bearing-down efforts do not add to the risk of rupture because they do not add to the stress on the uterine wall. (S-26)

A client with a 38-week twin gestation is admitted to a birthing center in early labor. One of the fetuses is a breech presentation. Which intervention is least appropriate in planning the nursing care of this client? 1. Measure fundal height. 2. Attach electronic fetal monitoring. 3. Prepare the client for a possible cesarean section. 4. Visually examine the perineum and vaginal opening.

**1. Measure fundal height. **Rationale: Measuring fundal height is least appropriate because it should be measured at each antepartum clinic visit, not in the intrapartum period. All other options are priorities. Intrapartum management and assessment require careful attention to maternal and fetal status. The fetuses should be monitored by dual electronic fetal monitoring, and any signs of distress must be reported to the health care provider. A cesarean section may be necessary if a fetus is breech. The nurse should examine the perineum and vaginal opening visually for signs of the cord, which sometimes prolapses through the cervix. (S-35)

The maternity nurse is caring for a client with abruptio placentae and is monitoring her for disseminated intravascular coagulation (DIC). Which assessment findings are most likely associated with disseminated intravascular coagulation? Select all that apply. 1. Petechiae 2. Hematuria 3. Increased platelet count 4. Prolonged clotting times 5. Oozing from injection sites 5. Swelling of the calf of 1 leg

**1. Petechiae; 2. Hematuria; 4. Prolonged clotting times; 5. Oozing from injection sites **Rationale: DIC is a state of diffuse clotting in which clotting factors are consumed, leading to widespread bleeding. -- Platelets are decreased because they are consumed by the process. -- Coagulation studies show no clot formation (and are thus normal to prolonged), and fibrin plugs may clog the microvasculature diffusely rather than in an isolated area. -- The presence of petechiae, hematuria, and oozing from injection sites are signs associated with DIC. -- Swelling and pain in the calf of 1 leg are more likely to be associated with thrombophlebitis. (S-25)

The nurse is providing emergency measures to a client in labor who has been diagnosed with a prolapsed cord. The mother becomes anxious and frightened and says to the nurse, "Why are all of these people in here? Is my baby going to be all right?" Which client problem is most appropriate to address at this time? 1. The client's fear 2. The client's fatigue 3. The client's inability to control the situation 4. The client's inability to cope with the situation

**1. The client's fear **Rationale: The mother is anxious and frightened, and the most appropriate problem to address for the client at this time is fear. There are no data in the question to support a client problem with fatigue, inability to control the situation or inability to cope with the situation. These problems may be considered for this client at some point during the hospitalization experience. (S-24)

The nurse is caring for a client in labor who is receiving oxytocin by intravenous infusion to stimulate uterine contractions. Which assessment finding should indicate to the nurse that the infusion needs to be discontinued? 1. Increased urinary output 2. A fetal heart rate of 90 beats/minute 3 Contractions occurring within a 10-minute period 4. Adequate resting tone of the uterus palpated between contractions

**2. A fetal heart rate of 90 beats/minute **Rationale: A normal fetal heart rate is 110 to 160 beats/minute. -- Bradycardia or late or variable decelerations indicate fetal distress and the need to discontinue the oxytocin. -- Increased urinary output is unrelated to the use of oxytocin. -- The goal of labor augmentation is to achieve 3 good-quality contractions (appropriate intensity and duration) in a 10-minute period. -- The uterus should return to resting tone between contractions, and there should be no evidence of fetal distress. (S-31)

The nurse is admitting a pregnant client to the labor room and attaches an external electronic fetal monitor to the client's abdomen. After attachment of the electronic fetal monitor, what is the next nursing action? 1. Identify the types of accelerations. 2. Assess the baseline fetal heart rate. 3. Determine the intensity of the contractions. 4. Determine the frequency of the contractions.

**2. Assess the baseline fetal heart rate. **Rationale: Assessing the baseline fetal heart rate is important so that abnormal variations of the baseline rate can be identified if they occur. The intensity of contractions is assessed by an internal fetal monitor, not an external fetal monitor. Options 1 and 4 are important to assess, but not as the first priority. Fetal heart rate is evaluated by assessing baseline and periodic changes. Periodic changes occur in response to the intermittent stress of uterine contractions and the baseline beat-to-beat variability of the fetal heart rate. (S-11)

The nurse is caring for a client who is experiencing a precipitous labor and is waiting for the health care provider to arrive. When the infant's head crowns, what instruction should the nurse give the client? 1. Bear down. 2. Breathe rapidly. 3. Hold your breath. 4. Push with each contraction.

**2. Breathe rapidly. **Rationale: During a precipitous labor [expulsion of the fetus within less than 3 hours of commencement of regular contractions], when the infant's head crowns the nurse instructs the client to breathe rapidly to decrease the urge to push. The client is not instructed to push or bear down. Holding the breath decreases the amount of oxygen to the mother and the fetus. (S-27)

The nurse prepares a plan of care for the client with preeclampsia and documents that if the client progresses from preeclampsia to eclampsia, the nurse should take which first action? 1. Administer oxygen by face mask. 2. Clear and maintain an open airway. 3. Administer magnesium sulfate intravenously. 4. Assess the blood pressure and fetal heart rate.

**2. Clear and maintain an open airway. **Rationale: The first action during a seizure (eclampsia) is to ensure a patent airway. All other options are actions that follow. (S-36)

The maternity nurse is preparing for the admission of a client in the third trimester of pregnancy who is experiencing vaginal bleeding and has a suspected diagnosis of placenta previa. The nurse reviews the health care provider's prescriptions and should *question* which prescription? 1. Prepare the client for an ultrasound. 2. Obtain equipment for a manual pelvic examination. 3. Prepare to draw a hemoglobin and hematocrit blood sample. 4. Obtain equipment for external electronic fetal heart rate monitoring.

**2. Obtain equipment for a manual pelvic examination. **Rationale: Placenta previa is an improperly implanted placenta in the lower uterine segment near or over the internal cervical os. Manual pelvic examinations are contraindicated when vaginal bleeding is apparent until a diagnosis is made and placenta previa is ruled out. --Digital examination of the cervix can lead to hemorrhage. --A diagnosis of placenta previa is made by ultrasound. --The hemoglobin and hematocrit levels are monitored, and external electronic fetal heart rate monitoring is initiated. Electronic fetal monitoring (external) is crucial in evaluating the status of the fetus, who is at risk for severe hypoxia. (S-17)

The nurse explains the purpose of effleurage to a client in early labor. Which statement should the nurse include in the explanation? 1. "It is the application of pressure to the sacrum to relieve a backache." 2. "It is a form of biofeedback to enhance bearing-down efforts during delivery." 3. "It is light stroking of the abdomen to facilitate relaxation during labor and provide tactile stimulation to the fetus." 4. "It is performed to stimulate uterine activity by contracting a specific muscle group while other parts of the body rest."

**3. "It is light stroking of the abdomen to facilitate relaxation during labor and provide tactile stimulation to the fetus." **Rationale: Effleurage is a specific type of cutaneous stimulation involving light stroking of the abdomen and is used before transition to promote relaxation and relieve mild to moderate pain. Effleurage also provides tactile stimulation to the fetus. (S-28)

Which assessment following an amniotomy should be conducted first? 1. Cervical dilation 2. Bladder distention 3. Fetal heart rate pattern 4. Maternal blood pressure

**3. Fetal heart rate pattern **Rationale: Fetal heart rate is assessed immediately after amniotomy to detect any changes that may indicate cord compression or prolapse. When the membranes are ruptured, minimal vaginal examinations would be done because of the risk of infection. Bladder distention or maternal blood pressure would not be the first thing to check after an amniotomy. (S-13)

A prenatal client with severe abdominal pain is admitted to the maternity unit. The nurse is monitoring the client closely because concealed bleeding is suspected. Which assessment findings indicate the presence of concealed bleeding? Select all that apply. 1. Back pain 2. Heavy vaginal bleeding 3. Increase in fundal height 4. Hard, boardlike abdomen 5. Persistent abdominal pain 6. Early deceleration on the fetal heart monitor

**3. Increase in fundal height; 4. Hard, boardlike abdomen; 5. Persistent abdominal pain **Rationale: The signs of concealed abdominal bleeding in a pregnant client include an increase in fundal height; hard, boardlike abdomen; persistent abdominal pain; late decelerations in fetal heart rate; and decreasing baseline variability. Back pain, heavy vaginal bleeding, and early deceleration on the fetal heart monitor are not specific signs of concealed bleeding.(S-38)

The nurse is reviewing the health care provider's (HCP's) prescriptions for a client admitted for premature rupture of the membranes. Gestational age of the fetus is determined to be 37 weeks. Which prescription should the nurse question? 1. Monitor fetal heart rate continuously. 2. Monitor maternal vital signs frequently. 3. Perform a vaginal examination every shift. 4. Administer an antibiotic per HCP prescription and per agency protocol.

**3. Perform a vaginal examination every shift. **Rationale: Vaginal examinations should not be done routinely on a client with premature rupture of the membranes because of the risk of infection. The nurse would expect to monitor fetal heart rate, monitor maternal vital signs, and administer an antibiotic. (S-21)

The nurse is reviewing true and false labor signs with a multiparous client. The nurse determines that the client understands the signs of true labor if she makes which statement? 1. "I won't be in labor until my baby drops." 2. "My contractions will be felt in my abdominal area." 3. "My contractions will not be as painful if I walk around." 4. "My contractions will increase in duration and intensity."

**4. "My contractions will increase in duration and intensity." **Rationale: True labor is present when contractions increase in duration and intensity. Lightening or dropping leads to engagement (presenting part reaches the level of the ischial spine) and occurs when the fetus descends into the pelvis about 2 weeks before delivery. Contractions felt in the abdominal area and contractions that ease with walking are signs of false labor. (S-12)

The nurse is performing an assessment on a client diagnosed with placenta previa. Which assessment findings should the nurse expect to note? Select all that apply. 1. Uterine rigidity 2. Uterine tenderness 3. Severe abdominal pain 4. Bright red vaginal bleeding 5. Soft, relaxed, nontender uterus 6. Fundal height may be greater than expected for gestational age

**4. Bright red vaginal bleeding; 5. Soft, relaxed, nontender uterus; 6. Fundal height may be greater than expected for gestational age **Rationale: -- Placenta previa is an improperly implanted placenta in the lower uterine segment near or over the internal cervical os. Painless, bright red vaginal bleeding in the second or third trimester of pregnancy is a sign of placenta previa. The client has a soft, relaxed, nontender uterus, and fundal height may be more than expected for gestational age. -- In abruptio placentae, severe abdominal pain is present. Uterine tenderness accompanies placental abruption. In addition, in abruptio placentae, the abdomen feels hard and boardlike on palpation as the blood penetrates the myometrium and causes uterine irritability. (S-23)

(ATI) A nurse in the labor and delivery unit is caring for a client in labor and applies an external fetal monitor and tocotransducer. The FHR is around 140 /min. Contractions are occurring every 8 min. and 30-40 sec. in duration. The nurse performs a vaginal exam and finds the cervix is 2 cm. dilated, 50% effaced, and the fetus is at -2 station. Which of the following stages and phases of labor is this client experiencing? A. First stage, latent phase B. First stage, active phase C. First stage, transition phase D. Second stage of labor

**A. First stage, latent phase **Rationale: In stage 1, latent phase, the cervix dilates from 0 to 3cm, and contraction duration ranges from 30-45 seconds. *Other answer choices rationales: B. In stage 1, active phase, the cervix dilates from 4-7cm and contraction duration ranges from 40-70 seconds. C. In stage 1, transition phase, the cervix dilates from 8 to 10 cm. and contraction duration ranges from 45-90 seconds. D. The second stage of labor consists of expulsion or delivey of fetus

(ATI) A nurse in the labor and delivery unit is completing an admission assessment for a client who is at 39 weeks gestation. The client reports that she has been leaking fluid from her vagina for 2 days. Which of the following conditions is the client at risk for developing? A. Cord prolapse B. Infection C. Postpartum hemorrhage D. Hydramnios

**B is correct. -- Infection **Rupture of membranes for longer than 24 hours prior to delivery increases the risk that infectious organisms can enter the vagina and eventually enter the uterus. **Rationales related to other answer choices: A. Cord prolapse occurs when there is a rush of fluid rather than a trickle. C. This risk is not increased with leaking membranes. D. This client is most likely to experience oligohydramnios [insufficient amniotic fluid] rather than hydramnios [excess amniotic fluid].

(ATI) A client experiences a large gush of fluid from her vagina while walking in the hallway at the birthing unit. Which of the following actions should the nurse take first? A. Check amniotic fluid for meconium B. Monitor FHR for distress C. Dry the client and make her comfortable D. Monitor uterine contractions

**B. Monitor FHR for distress **Rationale: The greatest risk to the client and fetus is umbilical cord prolapse leading to fetal distress following the rupture of membranes. The FIRST action by the nurse is to monitor FHR for clinical findings of distress. *Rationales related to other answer choices: A. This is not the first action, but is an action that should be done to check color, clarity, odor, and amount of amniotic fluid. C. This is not the first action, but is an action that should be done for client comfort. D. This is not the first action, but is an action that should be done to monitor client uterine contraction pattern after rupture of membranes.

(ATI.) A nurse is caring for a client who is in active labor and becomes nauseous and vomits. The client is very irritable and feels the urge to have a bowel movement. She states, "I've had enough. I can't do this any more. I want to go home right now!" Which of the following stages of labor is the client experiencing? A. Second stage B. Fourth stage C. Transition phase D. Latent phase

**C. is correct. - Transition phase **The transition phase of labor occurs when the client becomes irritable, feels rectal pressure similar to the need to have a bowel movement, and can become nauseous with emesis. **Rationales related to other answer choices: A. The second stage of labor occurs with the expulsion of the baby B. The fourth stage of labor is the recovery period, following the delivery of the placenta D. The latent phase of labor occurs in stage 1 and coincides with mild contractions. The client is usually relaxed, talkative, and eager for labor to progress.

The nurse is assisting a client undergoing induction of labor at 41 weeks of gestation. The client's contractions are moderate and occurring every 2 to 3 minutes, with a duration of 60 seconds. An internal fetal heart rate monitor is in place. The baseline fetal heart rate has been 120 to 122 beats/minute for the past hour. What is the priority nursing action?

**Discontinue the infusion of oxytocin. **Rationale: The PRIORITY nursing action is to stop the infusion of oxytocin. Oxytocin can cause forceful uterine contractions and decrease oxygenation to the placenta, resulting in decreased variability. After stopping the oxytocin, the nurse should reposition the laboring mother. --Notifying the health care provider, applying oxygen, and increasing the rate of the intravenous (IV) fluid (the solution without the oxytocin) are also actions that are indicated in this situation, but not the PRIORITY action. Contacting the client's primary support person(s) is not the priority action at this time. (S-15)

The nurse in the labor room is caring for a client in the first stage of labor. When monitoring the fetal patterns, the nurse notes an early deceleration of the fetal heart rate (FHR) on the monitor strip. Which is the appropriate nursing action?

**Document the findings and continue to monitor fetal patterns. Rationale: Early deceleration of the FHR refers to a gradual decrease in the heart rate, followed by a return to baseline, in response to compression of the fetal head. It is a normal and benign finding. Because early decelerations are considered benign, interventions are not necessary.

The nurse is caring for a client in labor and is monitoring the fetal heart rate patterns. The nurse notes the presence of episodic accelerations on the electronic fetal monitor tracing. Which action is most appropriate?

**Document the findings and tell the mother that the pattern on the monitor indicates fetal well-being. **Rationale: Accelerations are transient increases in the fetal heart rate that often accompany contractions or are caused by fetal movement. Episodic (occurs without a contraction) accelerations are thought to be a sign of fetal well-being and adequate oxygen reserve. (S-10)

A client in labor is dilated 10 cm. At this point in the labor process, at least how often should the nurse assess and document the fetal heart rate?

**Every 15 minutes **Rationale: The second stage of labor begins when the cervix is dilated completely (10 cm). Maternal pulse, blood pressure, and fetal heart rate are assessed every 5 to 15 minutes, depending on agency protocol; some agency protocols recommend assessment after each contraction. (S-29)

The nurse is caring for a client in labor and prepares to auscultate the fetal heart rate (FHR) by using a Doppler ultrasound device. Which action should the nurse take to determine fetal heart sounds accurately?

**Palpating the maternal radial pulse while listening to the FHR **Rationale: The nurse should simultaneously palpate the maternal radial or carotid pulse and auscultate the FHR to differentiate between the two. If the fetal and maternal heart rates are similar, the nurse may mistake the maternal heart rate for the FHR. (S-30)

The nurse in a birthing room is monitoring a client with dysfunctional labor for signs of fetal or maternal compromise. Which assessment finding should alert the nurse to a compromise?

**Persistent nonreassuring fetal heart rate **Rationale: Signs of fetal or maternal compromise include a persistent, nonreassuring fetal heart rate, fetal acidosis, and the passage of meconium. -- Maternal fatigue and infection can occur if the labor is prolonged, but do not indicate fetal or maternal compromise. -- Coordinated uterine contractions and progressive changes in the cervix are a reassuring pattern in labor. (S-19)

The nurse assists in the vaginal delivery of a newborn infant. After the delivery, the nurse observes the umbilical cord lengthen and a spurt of blood from the vagina. The nurse documents these observations as signs of which condition?

**Placental separation **Rationale: As the placenta separates, it settles downward into the lower uterine segment. The umbilical cord lengthens, and a sudden trickle or spurt of blood appears. (S-33)

During the intrapartum period, the nurse is caring for a client with sickle cell disease. The nurse ensures that the client receives adequate intravenous fluid intake and oxygen consumption to achieve which outcome?

**Prevent dehydration and hypoxemia. **Rationale: A variety of conditions, including dehydration, hypoxemia, infection, and exertion, can stimulate the sickling process during the intrapartum period. Maintaining adequate intravenous fluid intake and the administration of oxygen via face mask will help to ensure a safe environment for maternal and fetal health during labor. (S-34)

The nurse in a labor room is preparing to care for a client with hypertonic uterine contractions. The nurse is told that the client is experiencing uncoordinated contractions that are erratic in their frequency, duration, and intensity. What is the priority nursing action? 1. Provide pain relief measures. 2. Prepare the client for an amniotomy. 3. Promote ambulation every 30 minutes. 4. Monitor the oxytocin infusion closely.

**Provide pain relief measures. **Rationale: Hypertonic uterine contractions are painful, occur frequently, and are uncoordinated. Management of hypertonic labor depends on the cause. Relief of pain is the primary intervention to promote a normal labor pattern. -- An amniotomy and oxytocin infusion are not treatment measures for hypertonic contractions; however, these treatments may be used in clients with hypotonic dysfunction. -- A client with hypertonic uterine contractions would not be encouraged to ambulate every 30 minutes, but would be encouraged to rest. (S-20)

The nurse has been working with a laboring client and notes that she has been pushing effectively for 1 hour. What is the client's primary physiological need at this time?

**Rest between contractions **Rationale: The birth process expends a great deal of energy, particularly during the transition stage. *Encouraging rest between contractions conserves maternal energy, facilitating voluntary pushing efforts with contractions. Uteroplacental perfusion also is enhanced, which promotes fetal tolerance of the stress of labor. --Ambulation is encouraged during early labor. Ice chips should be provided. Changing positions frequently is not the primary physiological need. Food and fluids are likely to be withheld at this time. (S-14)

The nurse caring for a client who is receiving oxytocin (Pitocin) for the induction of labor notes a nonreassuring fetal heart rate (FHR) pattern on the fetal monitor. On the basis of this finding which is the nurse's priority action?

**Stop the oxytocin infusion. Rationale: Oxytocin stimulates uterine contractions and is used to induce labor. If uterine hypertonicity or a nonreassuring FHR pattern occurs, the nurse needs to intervene to reduce uterine activity and increase fetal oxygenation. The oxytocin infusion is stopped, the client is placed in a side-lying position, and oxygen by face mask at 8 to 10 L/min is administered. The health care provider is notified. The nurse would monitor the client's blood pressure and intake and output; however, the nurse would first stop the infusion.

A client in labor is transported to the delivery room and prepared for a cesarean delivery. After the client is transferred to the delivery room table, the nurse should place the client in which position?

**Supine position with a wedge under the right hip (S-8) *Rationale: Vena cava and descending aorta compression by the pregnant uterus impedes blood return from the lower trunk and extremities. This leads to decreasing cardiac return, cardiac output, and blood flow to the uterus and subsequently the fetus. The best position to prevent this would be side-lying, with the uterus displaced off the abdominal vessels. Positioning for abdominal surgery necessitates a supine position, however; a wedge placed under the right hip provides displacement of the uterus. Trendelenburg's position places pressure from the pregnant uterus on the diaphragm and lungs, decreasing respiratory capacity and oxygenation. A prone or semi Fowler's position is not practical for this type of abdominal surgery.

The advantages of using spinal anesthesia for delivery of a fetus include which? Select all that apply.

--Ease of administration --Absence of fetal hypoxia --Immediate onset of anesthesia *Typically used spinal anesthesias: Bupivacaine hydrochloride, Ropivacaine hydrochloride

CNS dysfunction in preeclampsia

--affects the ability to perform controlled, coordinated movement --causes abnormal changes in muscle tone, balance, posture, reflexive movements affects sensation --results b/c of increased ICP and commonly causes vision changes, cortical blindness, severe persistent headache, altered mental status (AMS), or seizures / stroke

Postpartum use of magnesium sulfate (MgSO4) in preeclampsia clients:

--continue 12-24 hours --Monitor urine output --Anti-hypertensives --Monitor for lab abnormalities every 4-6 hours until recovery --No NSAIDs if: uncontrolled HTN, poor renal function, oliguria, or low platelets --Follow up with HCP in 1 week

The nurse is collecting data from a client who has been diagnosed with placenta previa. Which findings should the nurse expect to note? Select all that apply.

-Bright red vaginal bleeding -Soft, relaxed, nontender uterus

The nurse in the labor room is caring for a client in the active stage of the first phase of labor. The nurse is assessing the fetal pattern and notes a late deceleration on the monitor strip. What is the "most appropriate" nursing action? 1. Administer oxygen via face mask. 2. Place the mother in a supine position. 3. Increase the rate of the oxytocin intravenous infusion. 4. Document the findings and continue to monitor the fetal pattern.

1 is the correct answer - administer oxygen via face mask. **Rationale: Late decelerations are due to uteroplacental insufficiency and occur because of decreased blood flow and oxygen to the fetus during uterine contractions. Hypoxemia results; oxygen at 8-10 L / minute via face mask is necessary. -- The supine position is avoided because it decreases uterine blood flow to the fetus. The client should be turned on her side to displace pressure of the gravid uterus on the inferior vena cava. Oxytocin is discontinued when late decelerations occur because it would further depress oxygenation. (S-270-3)

Signs and symptoms indicating preeclampsia:

1. Onset after 20 weeks 2. Systolic BP equal to or greater than 140 and/or diastolic BP equal to or greater than 90 -- at least 2 separate readings taken 4 hours apart 3. Proteinuria s/s 300mg per 24 hour or protein/creatinine ratio of 0.3 mg/dl or dipstick reading of 1+ [only use dip stick if other measurement methods not available ] 4. If no protein - can still be preeclampsia if one of the following is present: platelet count is less than 100,000; Serum creatinine is 1.1 or doubling of creatinine (in pregnancy b/c of increased plasma and increased kidney function/GFR the normal value should be about 0.5); Liver enzymes elevated to twice normal reference values; pulmonary edema; visual disturbances, headache, epigastric/RUQ pain

The nurse in a maternity unit is reviewing the clients' records. Which clients should the nurse identify as being at the most risk for developing disseminated intravascular coagulation (DIC)? Select all that apply. 1. A primigravida with mild preeclampsia 2. A primigravida who delivered a 10-lb infant 3 hours ago 3. A gravida II who has just been diagnosed with dead fetus syndrome 4. A gravida IV who delivered 8 hours ago and has lost 500 mL of blood 5. A primigravida at 29 weeks of gestation who was recently diagnosed with severe preeclampsia

3. A gravida II who has just been diagnosed with dead fetus syndrome 5. A primigravida at 29 weeks of gestation who was recently diagnosed with severe preeclampsia Rationale: In a pregnant client, DIC is a condition in which the clotting cascade is activated, resulting in the formation of clots in the microcirculation. Dead fetus syndrome is considered a risk factor for DIC. Severe preeclampsia is considered a risk factor for DIC; a mild case is not. Delivering a large newborn is not considered a risk factor for DIC. Hemorrhage is a risk factor for DIC; however, a loss of 500 mL is not considered hemorrhage. **See power point for mL considered hemorrhage.

(ATI) A nurse in the labor and delivery unit receives a phone call from a client who reports that her contractions started about 2 hours ago, did not go away when she had two glasses of water and rested, and became stronger since she started walking. Her contractions occur every 10 minutes and last about 30 seconds. She hasn't had any fluid leak from her vagina. However, she saw some blood when she wiped after voiding. Based on this report, which of the following clinical findings should the nurse recognize that the client is experiencing? A. Braxton Hicks contractions B. Rupture of membranes C. Fetal descent D. True contractions

D. True contractions Rationale: True contractions do not go away with hydration or walking. They are regular in frequency, duration, and intensity; and become stronger with walking. Other answer choices: A. Braxton Hicks contractions decrease with hydration and walking. B. Rupture of membranes would be indicated by the presence of a gush of fluid unrelated to the client's activity. C. Fetal descent is the downward movement of the fetus in the birth canal and cannot be evaluated based on client's report.

The nurse is asked to assist the primary health care provider in performing Leopold's maneuvers on a client. Which nursing intervention should be implemented before this procedure is performed?

Have the client empty her bladder.

(ATI) What are the maternal characteristics associated with the active phase of the 1st stage of labor?

Primagravida active phase usually lasts about 3 hours. Multigravida active phase usually lasts about 2 hours. Cervix dilation from 4 to 7 cm. Contractions are more regular and moderate to strong, occurring every 3-5 minutes, with a duration of 40-70 seconds. The active phase is usually a time of rapid dilation and effacement, with some fetal descent. The mom has feelings of helplessness. Mom's anxiety and restlessness increase as the contractions become stronger.

Pathophysiology of severe preeclampsia:

The characteristic features are systemic vasoconstriction, vasospasm due to decreasing blood flow to tissues and cells, endothelial damage causing escape of intravascular volume, decrease in circulating plasma volume, platelet aggregation resulting in microvascular obstruction. These characteristics can lead to progressive organ damage: brain [ headache, vision change, seizure], liver [ epigastric-abdominal pain, nausea], and kidney [ decreased urine output, renal failure].

Leopold's maneuvers will be performed on a pregnant client. The client asks the nurse about the procedure. Which information should the nurse provide to the client about Leopold's maneuvers?

The maneuvers are a systematic method for palpating the fetus through the maternal abdominal wall. They are a way to determine the position of a fetus inside the woman's uterus. **NOTES: Leopold's Maneuvers Leopold's maneuvers are four specific steps in palpating the uterus through the abdomen in order to determine the lie and presentation of the fetus. In summary the steps are : *Step 1. The top of the uterus (fundus) is felt (palpated) to establish which end of the fetus (fetal pole) is in the upper part of the uterus. If either the head or breech (buttocks) of the fetus are in the fundus then the fetus is in vertical lie. Otherwise the fetus is most likely in transverse lie. *Step 2. Firm pressure is applied to the sides of the abdomen to establish the location of the spine and extremities (small parts). *Step 3. Using the thumb and fingers of one hand the lower abdomen is grasped just above the pubic symphysis to establish if the presenting part is engaged. If not engaged a movable body part will be felt. The presenting part is the part of the fetus that is felt to be in closest proximity to the birth canal. *Step 4. Facing the maternal feet the tips of the fingers of each hand are used to apply deep pressure in the direction of the axis of the pelvic outlet. If the head presents, one hand is arrested sooner than the other by a rounded body (the cephalic prominence) while the other hand descends deeply into the pelvis. If the cephalic prominence is on the same side as the small parts, then the fetus is in vertex presentation. If the cephalic prominence is on the same side as the back , then the head is extended and the fetus is in face presentation.

The nurse is assigned to assist with caring for a client with abruptio placentae who is experiencing vaginal bleeding. The nurse collects data from the client, knowing that abruptio placentae is accompanied by which additional finding?

Uterine tenderness accompanies abruptio placentae, especially with a central abruption and trapped blood behind the placenta. The abdomen will feel hard and boardlike on palpation as the blood penetrates the myometrium and causes uterine irritability. A sustained tetanic contraction can occur if the client is in labor and the uterine muscle cannot relax.

The initial side effect of an epidural anesthesia is fetal bradycardia A. True B. False

A. True- due the vasodilation and hypotension caused by the epidural, the fetus is more likely to have a nonreassuring fetal strip.

What does absent or decreased variability on FHR monitor strip indicate? (Lecture / powerpoint)

Absence or decreased variability or smooth flat baseline is a signal of fetal compromise. [Etiologies of decreased variability: Fetal metabolic acidosis, CNS depressants, fetal sleep cycles, congenital anomalies, prematurity, fetal tachycardia, preexisting neurologic abnormality, betamethasone. http://perinatology.com/Fetal%20Monitoring/Intrapartum%20Monitoring.htm ]

What are some factors associated with fetal heart rate (FHR) regulation? (Lecture / power point)

Autonomic nervous system, baroreceptors, chemoreceptors, adrenal gland, and central nervous system are all associated with FHR regulation.

Cardinal movement that occurs as the fetal shoulders engage and descend through the pelvis is termed ______. A. Internal rotation B. External rotation

B- External rotation (p. 390)

Cardinal movement that allows the smallest diameter of the head to pass through the pelvis is__________________. A. Flexion B. Internal rotation C. Extension

B- Flexion (p.388)

___ = Endogenous opiods secreted by the pituitary gland that act on the central and peripheral nervous systems to reduce pain; their level increases during pregnancy and birth, thereby enhancing a woman's ability to tolerate acute pain and reducing anxiety.

Beta endorphins

The nurse is assigned to work in the delivery room and is assisting with caring for a client who has just delivered a newborn. The nurse is monitoring for signs of placental separation knowing that which indicates that the placenta has separated?

Changes in the shape of the uterus

A primigravida's membranes rupture spontaneously. Which action should the nurse take first?

Checks the fetal heart rate

The nurse is caring for a client who is in labor. The nurse rechecks the client's blood pressure and notes that it has dropped. To decrease the incidence of supine hypotension, the nurse should encourage the client to remain in which position?

Client should remain in a side-lying position; preferably left lateral

The nurse is assigned to assist with caring for a client who is being admitted to the birthing center in early labor. On admission, which action should the nurse take initially?

Determine the maternal and fetal vital signs.

Effleurage is ___.

Effleurage is a type of massage using light pressure applied over a wide area of the body. --Light effleurage promotes relaxation, alleviates pain and encourages sleep. --Deep effleurage improves circulation while stretching and relaxing tense muscles.

A client is scheduled to have an elective cesarean delivery. How should the nurse allay the client's feelings of anxiety?

Encouraging the client to discuss her concerns and desires regarding anesthesia options

___ = Theory of pain based on the premise that pain sensations travel along sensory nerve pathways to the brain, but only a limited number of sensations or messages can travel through these nerve pathways at one time; using distraction techniques such as massage, music, focal points, or imagery the capacity of nerve pathways are diminished.

Gate control theory of pain

What is fetal scalp stimulation? (Lecture / powerpoint)

Gentle stoking or massaging of fetal scalp for 15 seconds during vaginal examination. Assess fetal tracing for signs of accelerations of 15 bpm for 15 sec. which is a sign of fetal well-being.

Severe Pre-Eclampsia (HELLP)

H- hemolysis (the breakdown of red blood cells) EL- elevated liver enzymes/function tests LP- low platelet count **Most often HELLP develops during the third trimester of pregnancy (between 26 to 40 weeks gestation). Sometimes it develops in the week after the baby is born.

What are the effects of vasospasm on the fetus? (Lecture / powerpoint/ PIH - Pregnancy Induced Hypertension - aka gestational hypertension)

Increased BP is due to generalized arteriolar cyclic vasospasms which decrease the diameter of blood vessels. It results in increased peripheral resistance and impeded blood flow. Fluid is forced out of the vessels causing hemoconcentration leading to increased blood viscosity = Increased hematocrit. It is very difficult to circulate thick blood. This causes endothelial cell damage, intravascular fluid redistribution, and decreased organ perfusion which leads to multi-system failure. Decreased maternal perfusion leads to decreased oxygenation and perfusion of fetus.

___ = Paced breathing technique during which a woman breathes at approximately 6-8 breaths per minute (half the normal breath rate).

Lamaze

What is the effect of vasospasm on the brain? (Lecture / powerpoint/ PIH)

Limits oxygenation and perfusion of the brain that can result in decreased LOC, headache, hyperreflexia, convulsions

Lumbar epidural analgesia is ___. (Lecture / powerpoint)

Lumbar epidural analgesia is the most effective pharmacologic pain relief method for labor currently available Pain of uterine contractions and birth can be relieved by injecting a local anesthetic agent into the epidural space Can be used for both vaginal and cesarean births.

2. Continuation of Pitty Pitocin story: Pitocin is a stimulant, so as Pitty watches the ship sink, her blood pressure ELEVATES. Just as the sinking ship takes in all that salty water, poor Pitty is left holding the excess fluid in her body, so the nurse should ___ (Lecture handout)

Monitor intake and output

Adverse affects of prolonged pain:

PHYSIOLOGIC EFFECTS: *Increase in metabolic rate and oxygen demand *Increase in production of catecholamines, cortisol, glucagon *Less oxygen/waste exchange for fetus PSYCHOLOGICAL EFFECTS: *Difficulty interacting with the infant *Unpleasant memories *Partner may feel inadequate

___ = Level of pain a person is willing to endure.

Pain threshold

The client is admitted to the labor suite complaining of painless vaginal bleeding. The nurse assists with the examination of the client, knowing that which routine labor procedure is contraindicated?

Painless vaginal bleeding is a sign of possible placenta previa. Digital examination of the cervix is contraindicated because it can lead to maternal and fetal hemorrhage. Painless bright red vaginal bleeding during the second or third trimester of pregnancy is a sign of placenta previa. The client will have a soft and relaxed nontender uterus.

___ = Pain in labor and birth that originates in the uterus and radiates to the abdominal wall, lumbosacral area of the back, iliac crests, gluteal area, and down the thighs.

Referred

4. Continuation of Pitty Pitocin story: The cardiac arrhythmias and the fetal heart irregularities are so upsetting to Pitty that she begins to feel NAUSEA and she VOMITS. What should her nurse do? (Lecture handout)

STOP PITOCIN !!

___ = pain that dominated during the second stage of labor; it results from stretching and distention of perineal tissues and the pelvic floor to allow passage of the fetus, from distention and traction on the peritoneum and uterocervical supports during contractions, and from lacerations of soft tissues.

Somatic

The client who is being prepared for a cesarean delivery is brought to the delivery room. To maintain the optimal perfusion of oxygenated blood to the fetus, the nurse should place the client in which position?

Supine position with a wedge under the right hip

(ATI) What are the maternal characteristics associated with the 2nd stage of labor?

The 2nd stage of labor typically lasts 30 minutes to 2 hours for primagravida; and 5-30 minutes for multigravida. The mom's cervix is fully dilated at 10 cm. Contractions progress to intense contractions every 1 to 2 minutes. This stage's pushing results in birth of baby.

The nurse is reviewing the record of a client in the labor room and notes that the health care provider has documented that the fetal presenting part is at the -1 station. This documented finding indicates that the fetal presenting part is located at which area? Click on the image to indicate your answer.

The correct answer is 3, the first segment that is above the solid ischial spine line. (S-272-5)

How is internal FHR monitoring done? (Lecture / power point)

The spiral electrode is attached to the fetal scalp. Wires that extend from the spiral electrode are attached to a leg plate and then attached to an electronic fetal monitor.

Severe preeclampsia

These symptoms may appear suddenly. BP readings increase; 160/110 mm Hg or higher on two separate occasions 6 hours apart with pregnant woman on bed rest are common. Edema becomes increasingly obvious and may be observed in the face, the hands, the sacral area, the abdomen, and throughout the lower extremities. Weight increases dramatically. The woman may gain as much as two pounds in a matter of a few days or a week. Urine testing for albumin shows 3+ and 4+ readings. The urinary output is less than 500 mL./ 24 hr.

(ATI) What are the maternal characteristics associated with the transition phase of the 1st stage of labor?

Transition phase of 1st stage of labor usually takes about 20-40 minutes. Cervix dilates from 8cm to complete dilation of 10cm. Contractions are strong to very strong and occur at a frequency of every 2-3 minutes, with a duration of 45-90 seconds. The mom feels tired, restless, and irritable. She feels out of control and often states that she cannot continue. Mom has strong urge to push and may experience nausea and vomiting. Mom feels increasing rectal pressure, has increasing bloody show. **This phase is often the most difficult part of labor.

____ = decreased blood flow to the uterus during a contraction leading to an oxygen deficit.

Uterine ischemia

___ = pain that predominates during the first stage of labor; it results from cervical changes, distention of the lower uterine segment, and uterine ischemia.

Visceral

Myasthenia gravis is ___.

an autoimmune disorder. **Myasthenia gravis (MG) is a long-term neuromuscular disease that leads to varying degrees of skeletal muscle weakness. The most commonly affected muscles are those of the eyes, face, and swallowing. It can result in double vision, drooping eyelids, trouble talking, and trouble walking.

If excessive uterine activity persists, the fetus can develop ___

metabolic acidosis and, if severe enough, brain injury and even death can occur.

Lumbar epidural analgesia is ___.

the most effective pharmacologic pain relief method for labor currently available Pain of uterine contractions and birth can be relieved by injecting a local anesthetic agent into the epidural space **Can be used for both vaginal and cesarean births

Stadol & Nubain are synthetic agonist/antagonists narcotic analgesia

use caution administering to drug addicted pt. Can be given IM & IV. Decreased respiratory depression and addictive potential

The nurse is assessing the fetal station during a vaginal examination. Which of the following structures should the nurse palpate? 1. sacral promontory 2. Ischial spines 3. Cervix 4. Symphysis pubis

(D-7) 2 is correct - Ischial spines "Station" is assessed by palpating the ischial spines.

Which statement, if made by the laboring client, most likely indicates that the client is in the second stage of labor?

**"I feel like I need to push." **Rationale: The second stage of labor begins when the cervix is completely dilated and ends with birth of the infant. At this time, the laboring woman typically experiences the desire to push. (S-65)

The nurse has created a plan of care for a client experiencing dystocia and includes several nursing actions in the plan of care. What is the priority nursing action? 1. Providing comfort measures 2. Monitoring the fetal heart rate 3. Changing the client's position frequently 4. Keeping the significant other informed of the progress of the labor

**2. Monitoring the fetal heart rate **Rationale: Dystocia is difficult labor that is prolonged or more painful than expected. The priority is to monitor the fetal heart rate. Although providing comfort measures, changing the client's position frequently, and keeping the significant other informed of the progress of the labor are components of the plan of care, the fetal status would be the priority. (S-22)

A prenatal client with vaginal bleeding is being admitted to the labor unit. The labor room nurse is performing the admission assessment and should suspect a diagnosis of placenta previa if which finding is noted? 1. Back pain 2. Abdominal pain 3. Painful vaginal bleeding 4. Painless vaginal bleeding

**4. Painless vaginal bleeding **Rationale: The classic sign of placenta previa is the sudden onset of painless vaginal bleeding. Painful vaginal bleeding, abdominal pain, and back pain identify signs and symptoms of abruptio placentae. (S-37)

Fetal distress is occurring with a woman in labor. As the nurse prepares her for a cesarean birth, what other intervention should the nurse implement?

**Administer oxygen at 8 to 10 L/min via face mask. **Rationale: Oxygen is administered at 8 to 10 L/min via face mask to optimize oxygenation of the circulating blood volume. Oxytocin stimulates the uterus and is discontinued if fetal heart rate patterns change for any reason. The IV infusion should be increased, not decreased, so as to increase the maternal blood volume. The woman's position should be lateral with legs raised to increase maternal blood volume and improve the maternal vascular system. (S-52)

The goal for a woman with partial premature separation of the placenta is: "The woman will not exhibit signs of fetal distress." Which outcome, documented by the nurse, indicates that this goal has been achieved?

**Moderate variability present **Rationale: Reassuring signs in the fetal heart tracing include an FHR of 110 to 160 beats/minute, accelerations of the FHR, no variable decelerations, and the presence of moderate variability. The moderate variability [6-25 bpm] indicates that the fetus is able to make the necessary adjustments to the stresses of the labor. Variable decelerations indicate cord compression. (S-45)

The nurse is monitoring a client in active labor and notes that the client is having contractions every 3 minutes that last 45 seconds. The nurse notes that the fetal heart rate between contractions is 100 beats/minute. Which nursing action is most appropriate?

**Notify the health care provider (HCP). *Rationale: A normal fetal heart rate is 110 to 160 beats/minute, and the fetal heart rate should be within this range between contractions. Fetal bradycardia between contractions may indicate the need for immediate medical management, and the HCP or nurse-midwife needs to be notified. Options 2, 3, and 4 are inappropriate nursing actions in this situation and delay necessary intervention. (S-9)

On assessment of the fetal heart rate (FHR) of a laboring woman, the nurse discovers decelerations that have a gradual onset, last longer than 30 seconds, and return to the baseline rate with the completion of each contraction. The nurse plans care, knowing that this identifies which category of decelerations?

**Periodic, early decelerations that indicate fetal head compression **Rationale: An early deceleration is described as a visually apparent gradual decrease of the FHR with a gradual return to the FHR baseline. **Notes: Late decelerations do not return to the FHR baseline until after the uterine contraction is over. Variable decelerations are defined as having a rapid onset of less than 30 seconds with a rapid return to FHR baseline. Early decelerations are caused by fetal head compression, resulting from uterine contractions, vaginal examination, or fundal pressure. (S-57)

The nurse is monitoring a client with dysfunctional labor for signs of fetal or maternal compromise. Which finding should alert the nurse to a compromise?

**The passage of meconium **Rationale: Signs of fetal or maternal compromise include a persistent, nonreassuring fetal heart rate; fetal acidosis; and the passage of meconium (S-73)

What do the FHR monitor tachycardia or bradycardia patterns indicate? (Lecture / power point)

*Tachycardia is baseline above 160 bpm and is related to maternal fever, fetal hypoxia, intrauterine infection, and drugs. *Bradycardia is baseline below 100 and is related to profound hypoxia, anesthesia, and beta-adrenergic blocking drugs [-lol]

A pregnant client seen in the prenatal clinic tells the nurse that the iron supplement started 1 week ago is causing nausea, constipation, and heartburn and that she would like to stop taking the medication. The nurse responds by making which statement to the client?

--"These reactions are most prominent during initial therapy and lessen with continued use." Rationale: It is important that pregnant clients receive iron supplements because of the extra demands placed on maternal circulation by the fetus. (Saunders Maternal Meds)

(ATI) What are premonitory signs / maternal physiological changes that occur preceding labor?

1. Backache - constant, low, dull, caused by pelvic muscle relaxtion 2. Lightening - 3. Contractions - 1st Braxton Hicks that progress into regular contractions with significant strength 4. Increased vaginal discharge or bloody show - expulsion of cervical mucus plug, brownish-reddish indicating onset of cervical dilation & effacement 5. Energy burst - nesting instinct 6. GI changes - indigestion, nausea, vomiting

Interventions and nursing care for gestational hypertension / preeclampsia:

1. Decrease activities / promote bed rest: have client lay in left lateral position, provide quiet calm environment, restrict visitation to avoid stress and reduce vasospasms, sedative medications 2. Diet modifications: increase protein intake up to 70-80 gm/day to compensate for protein loss in urine, maintain balanced sodium intake, avoid caffeine to minimize vasoconstrictions. 3. Weigh self daily at same time of day b/c weight fluctuates throughout day 4. Keep record of fetal movement/kick count to note any reduced movement such as less than 10-12 movements per hour: call HCP if reduced b/c can indicate reduced fetal perfusion / well-being 5. Check urine for protein and amount of indicating preeclampsia

Causes of pain in Stage 2 of labor:

1. Distention of the vagina and perineum 2. Compression of the nerve ganglia in cervix & lower uterus 3. Pressure on urethra, bladder, rectum during fetal descent 4. Traction on and stretching of the perineum

Breathing patterns are taught to laboring women. Which breathing pattern should the nurse support for the woman and her coach during the latent phase of the first stage of labor if the couple had attended Lamaze classes? 1. Slow-paced breathing 2. Deep abdominal breathing 3. Modified-paced breathing 4. Patterned-paced breathing

1. Slow-paced breathing

Sources of pain in Stage 1 of labor:

1. Stretching of the cervix during dilation & effacement 2. Uterine Anoxia (an absence or deficiency of oxygen reaching the tissues; severe hypoxia) 3. Stretching of the uterine ligaments

Sources of labor pain:

*Tissue ischemia *Cervical dilation *Pressure and pulling on pelvic structures *Distention of the vagina and perineum

A client with severe preeclampsia is receiving intravenous magnesium sulfate. The nurse is reviewing the laboratory results and determines that which magnesium level is within the therapeutic range?

--5 mEq/L (2.5 mmol/L) Rationale: The therapeutic range for magnesium sulfate is 4 to 7 mEq/L (2 to 3.5 mmol/L)(Saunders Maternal Meds)

With preeclampsia, the kidneys are not filtrating properly. What are some significant lab work changes in serum chemistry r/t preeclampsia?

--Decreased urine creatinine clearance [80-130 mL/min.] --Increased BUN [12-30 mg/dL] --Increased serum creatinine [0.5-1.5 mg/dL] --Increased serum uric acid [3.5-6 mg/dL]

The nurse is preparing to administer exogenous surfactant to a premature infant who has respiratory distress syndrome. The nurse prepares to administer the medication by which route?

--Intratracheal Rationale: Respiratory distress syndrome is a serious lung disorder caused by immaturity and the inability to produce surfactant, resulting in hypoxia and acidosis. It is common in premature infants and may be due to lung immaturity as a result of surfactant deficiency. The mainstay of treatment is the administration of exogenous surfactant, which is administered by the intratracheal route. (Saunders Maternal Meds)

A client in preterm labor is being started on intravenous magnesium sulfate to stop the contractions. Several hours later, when the nurse is performing an assessment, the following data are obtained: blood pressure 110/66 mmHg, pulse 66 beats per minute, respirations 10 breaths per minute, and deep tendon reflexes absent. What should the nurse do next?

--Prepare to administer calcium gluconate as an antidote for magnesium toxicity. Rationale: The antidote for magnesium sulfate is calcium gluconate. This medication should be available if the client experiences magnesium toxicity. The respiratory rate and absence of deep tendon reflexes indicate magnesium toxicity and not the need for an increase in the rate of the medication. The client is not exhibiting signs and symptoms of preeclampsia, which might necessitate seizure precautions. (Saunders Maternal Meds)

The nurse is monitoring a client in labor. The nurse suspects umbilical cord compression if which is noted on the external monitor tracing during a contraction? 1. Variability 2. Accelerations 3. Early decelerations 4. Variable decelerations.

4 is correct - Variable decelerations. **Rationale: Variable decelerations occur if the umbilical cord becomes compressed, reducing blood flow between the placenta and the fetus. Variability refers to fluctuations in the baseline fetal heart rate. Accelerations are a reassuring sign and usually occur with fetal movement. Early decelerations result from pressure on the fetal head during a contraction. (S-274-7)

Mechanisms of Labor

Descent Engagement of the presenting part Flexion of the fetal head Internal rotation Extension of the fetal head External rotation Expulsion of the fetal shoulders and body

(ATI) What are the maternal characteristics associated with the 3rd stage of labor?

The 3rd stage of labor usually lasts about 5-30 minutes. It is the stage of labor where the placenta separation and expulsion occurs. SCHULTZ presentation: shiny fetal surface of placenta emerges 1st. DUNCAN presentation: dull maternal surface of placenta emerges 1st.

(ATI) What are the maternal characteristics associated with the 4th stage of labor?

The 4th stage of labor typically lasts 1-4 hours. It is a time of maternal stabilization of vital signs. Achievement of vital sign stabilization Lochia is scant to moderate rubra.

What kind of pain medication is used prior to an episiotomy or if tears are expected?

1% Lidocaine solution injected into the perineal tissue prior to an episiotomy or if tears are expected

What are some common methods of FHR monitoring? (Lecture / power point)

1. Intermittent auscultation by doppler (handheld ultrasound device at 10-12 weeks gestation) or fetoscope (sound amplifier device at 20 weeks gestation). 2. Continuous external monitoring 3. Continuous internal monitoring

Establishing a therapeutic relationship:

1. Making the family feel welcome: first impression, communicate interest, friendliness, caring, and competence, arranging for a culturally acceptable interpreter who is fluent in the woman's language 2. Determine family expectations about birth: birthing plans, support partners 3. Convey confidence in the woman's ability to give birth and her partner's ability to support her, provide reassurance that contractions are normal in active labor while helping her deal with them and watching for problems 4. Assigning a primary nurse: having one nurse is unrealistic, however, be limited as much as possible 5. Use Touch for Comfort: touch can communicate acceptance and reassurance and can provide physical and emotional comfort to many laboring woman 6. Incorporate a family's cultural practices into care as much as possible

The nurse is performing an assessment of a client who is scheduled for a cesarean delivery at 39 weeks gestation. Which assessment finding indicates the need to contact the HCP? 1. Hemoglobin of 11 g/dL 2. FHR of 180 beats/minute 3. Maternal pulse rate of 85 beats/minute 4. WBC count of 12,000

2 is correct. - FHR of 180 beats/minute **The reference fetal heart rate is 110-160 beats/minute. Outside this reference indicates fetal distress. *By full term hemoglobin reference range is 11-13 g/dL because of the hemodilution caused by an increase in plasma volume during pregnancy. *The maternal pulse rate during pregnancy increases 10-15 beats/minute over prepregnancy readings to facilitate increased cardiac output, oxygen transport, and kidney filtration. *WBC counts in a normal pregnancy begin to increase in the 2nd trimester and peak in the 3rd trimester with a reference range of 11,000-15,000, up to 18,000. -- During postpartum WBC may increase up to 25,000-30,000 because of increased leukocytosis that occurs during delivery. (S-271-4)

The nurse is caring for a client in labor and is monitoring the fetal heart rate patterns. The nurse notes the presence of episodic accelerations on the electronic fetal monitor tracing. Which action is most appropriate? 1. Notify the health care provider of the findings. 2. Reposition the mother and check the monitor for changes in the fetal tracing. 3. Take the mother's vital signs and tell the mother that bed rest is required to conserve oxygen. 4. Document the findings and tell the mother that the pattern on the monitor indicates fetal well-being.

4. Document the findings and tell the mother that the pattern on the monitor indicates fetal well-being.

The cardinal movement that facilitates the emergence of the fetal head is ____________. A. Flexion B. Extension C. External rotation

B-Extension- when the head emerges by extension, first the occiput, then the face, and finally the chin. (p.390)

The nurse would be careful to keep the patient flat following delivery with a pudendal block A. True B. False

B. False-A client is kept flat if adverse effect of spinal anesthesia due to cerebrospinal fluid leakage. A spinal headache is postural; it worsens when sitting up and disappear when lying down.

What is the definition of hypertension?

B/P = 140 / 90 if have no baseline. 1. 30 mm. Hg. systolic increase or a 15 mm. Hg. diastolic increase (two occasions four to six hours apart) 2. Increase in MAP > 20 mm.Hg over baseline or >105 mm. Hg. with no baseline

The nurse is assigned to care for a client who is in early labor. When collecting data from the client, which should the nurse check first?

Baseline FHR [fetal heart rate]

How are weight gain and edema with gestational hypertension related?

Decreased blood flow to the kidneys keeps pressure in kidney veins open causing a loss of plasma proteins and albumin. This leads to a decreased colloid osmotic pressure which allows fluid to shift from intravascular to extravascular, where the fluid then accumulates in the tissues. Increased angiotensin and aldosterone triggers retention of sodium and water, which then manifests as generalized edema and reduced kidney output.

Describe the four degrees of vaginal episiotomies.

Degrees of vaginal tears: 1st--First Degree-The smallest or most simple episiotomy, extending only through the vaginal mucosa. It does not involve the underlying tissues. 2nd--Second Degree-This is the most common type of episiotomy. It extends through the vaginal mucosa and into the submucosal tissues, but does not involve the rectal sphincter or mucosa. 3rd--Third Degree-A third degree episiotomy involves the vaginal mucosa, submucosal tissues, and a partial or complete transection of the anal sphincter muscle. 4th--Fourth Degree-The most severe type of episiotomy includes incision of the vaginal mucosa, submucosal tissues, and anal sphincter, and it also involves the lining of the rectum.

___ = Childbirth without fear, controlled breathing and conscious and progressive relaxation Keeps all muscles except uterus relaxed during contraction

Dick-Read method

Early deceleration pattern is related to ___. (Lecture / powerpoint)

Head compression. The onset and return of the deceleration coincide with the start and stop of maternal contractions. NO intervention is necessary. The nurse should monitor for any changes.

Late deceleration pattern is related to ___. (Lecture / powerpoint)

decreased uteroplacental perfusion. The fetal heart tones return to the baseline AFTER the end of maternal contraction.

Baseline variability is defined as ___

fluctuations in the fetal heart rate of more than 2 cycles per minute.

Normal pregnancy edema is found primarily in lower extremities, but the client with PIH/gestational hypertension clinical manifestations reveal ___

generalized, whole body, edema that appears suddenly, begins in lower extremities and moves upward. Pitting edema 1+ to 4+ and facial edema is a LATE sign. Weight gain is directly related to the accumulation of fluid in all tissues.

What are some nursing responsibilities related to electronic FHR monitoring? (Lecture / power point)

1. Placement of equipment 2. Teaching the client about its use 3. Notation of events on the strip 4. Evaluation of data from strip 5. Intervention as indicated by the data

The nurse is caring for a client in labor. Which assessment findings indicate to the nurse that the client is beginning the second stage of labor? Select all that apply. 1. The contractions are regular. 2. The membranes have ruptured. 3. The cervix is dilated completely. 4. The client begins to expel clear vaginal fluid. 5. The spontaneous urge to push is initiated from perineal pressure.

3. The cervix is dilated completely. 5. The spontaneous urge to push is initiated from perineal pressure. Rationale: The second stage of labor begins when the cervix is dilated completely and ends with birth of the neonate. The woman has a strong urge to push in stage 2 from perineal pressure.

(ATI) A manager of a labor and delivery unit is reviewing the procedure for vaginal examination with a group of newly hired nurses. What interventions should be included in this discussion? Describe assessment findings that can be determined by the procedure.

NURSING ACTIONS: (1) Explain procedure, and obtain client's permission for the examination. 2) Don sterile glove with antiseptic solution or soluble gel for lubrication. (3) Position client to avoid supine hypotension. (4) Provide privacy. (5) Cleanse the vulva or perineum as needed. (6) Insert index and middle finger into client's vagina. (7) Explain findings to client. (8) Document findings and report to HCP. OUTCOMES-EVALUATION: (1) Cervical dilation, effacement, and position. (2) Fetal presenting part, position, and station. (3) Status of membranes (4) Characteristics of amniotic fluid, if membranes are ruptured.

After a precipitous delivery, the nurse notes that the new mother is passive and only touches her newborn briefly with her fingertips. The nurse should do which to help the woman process what has happened?

Precipitous labor is labor that lasts 3 hours or less. Women who have experienced precipitous labor often describe feelings of disbelief that their labor progressed so rapidly. To assist the client to process what has happened, the best option is to support the client in her reaction to the newborn infant.

What are indications for FHR monitoring? (Lecture / power point)

Previous history of still birth, complications of pregnancy, induction of labor, preterm labor, nonreassuring fetal status / movement, and meconium staining of amniotic fluid are all related to causes of fetal hypoxia and indicate a need for FHR monitoring.

1. Pitocin - "Pitty Pitocin," this pregnant woman, is slow to begin active labor so the Doc decides to induce by using Pitocin. Watch for Major side effects! Visualize Pitty sitting in a row boat looking into a "PIT" watching the "TITANIC" sink into the ocean "OCIN." Use this mnemonic to remember side effects. (Lecture handout)

Side effects of oxytocin [Pitocin]. P-Pressure is elevated I-Intake and output needs to be monitored. T- Tetanic contractions-intense prolonged contractions lasting longer than 90-120 seconds. O-Oxygen decrease in fetus C-Cardiac arrhythmia I-Irregularity in fetal heart rate N-Nausea and vomiting

With preeclampsia, the main pathogenic factor

is not an increase in BP but poor perfusion as a result of vasospasm and reduced plasma volume. Arteriolar vasospasm diminishes the diameter of blood vessels, which impedes blood flow to all organs and increases BP Function in organs such as the placenta, kidneys, liver, and brain. *Plasma colloid osmotic pressure* decreases as serum albumin levels decrease. Intravascular volume is reduced as fluid moves out of the intravascular compartment, resulting in hemoconcentration, increased blood viscosity, and tissue edema. ( This increases risk for pulmonary edema) *Decreased liver perfusion* results in impaired liver function and elevated Liver enzyme. The women may complain of epigastric or right upper quadrant pain. Hemorrhagic necrosis in the liver can cause a sub-capsular hematoma is a life-threatening complication and a surgical emergency

The nurse is assigned to assist with caring for a client who has been admitted to the labor unit. The client is 9 cm dilated and is experiencing precipitous labor. Which is the priority nursing action?

keep the client in a side lying position

Test taking tip regarding a woman's prenatal record:

(D-1) It is a summary of the woman's history from the time she entered prenatal care until the record is sent to the labor room about 36 weeks gestation. If the woman has gained very little weight the baby may be small for gestational age.

The nurse enters a laboring client's room. The client is complaining of intense back pain with each contraction. The nurse concludes that the fetus is likely in which of the following positions? 1. Mentum anterior 2. Sacrum posterior 3. Occiput posterior 4. Scapula anterior

(D-11) 3. Occiput posterior RATIONALE:

When performing Leopold's maneuvers, the nurse notes that the fetus is in the left occiput anterior position. Which is the best position for the nurse to place a fetoscope to hear the fetal heartbeat? 1. Left upper quadrant 2. Right upper quadrant 3. Left lower quadrant 4. Right lower quadrant

(D-12) 3. Left lower quadrant RATIONALE:

On examination, it is noted that a full-term primipara in active labor is right occipitoanterior (ROA), 7 cm dilated, and+3 station. Which of the following should the nurse report to the physician? 1. Descent is progressing well. 2. Fetal head is not yet engaged. 3. Vaginal delivery is imminent. 4. External rotation is complete

(D-13) 1. Descent is progressing well. RATIONALE:

The childbirth educator is teaching a class to a group of pregnant teens. Which of the following strategies would promote learning by young women? 1. Avoiding the discussion of uncomfortable procedures like vaginal exams and blood tests. 2. Focusing the discussion on baby care rather than on labor and delivery. 3. Utilizing visual aids like movies and posters during the class. 4. Having the class at a location other than high school to reduce their embarassment.

(D-18) 3. Utilizing visual aids like movies and posters during the class. RATIONALE:

Advantages of epidural anesthesia

**Allows greater control over sensory level & results in a more gradual fall in arterial BP. **The woman remains alert and is more comfortable with ability to participate and interact with staff and family **Airway remains intact as only partial motor paralysis develops **Gastric emptying is not delayed **Less blood loss **Fetal complications are rare with rapid absorption or marked maternal hypotension ( initial dosing and re-bolusing)

The nurse is caring for a client in the transition phase of the first stage of labor. The client is experiencing uterine contractions every 2 minutes and she cries out in pain with each contraction. What is the nurse's best interpretation of this client's behavior?

**Fear of losing control **Rationale: Pain, helplessness, panicking, and fear of losing control are possible behaviors in the transition phase of the first stage of labor. (S-51)

The nurse is assigned to care for a client with hypotonic uterine dysfunction and signs of a slowing labor. The nurse is reviewing the health care provider's prescriptions and should expect to note which prescribed treatment for this condition?

**Oxytocin infusion **Rationale: Therapeutic management for hypotonic uterine dysfunction includes oxytocin augmentation and amniotomy to stimulate a labor that slows. A cesarean birth will be performed if no progress in labor occurs. (S-61)

An opioid analgesic is administered to a client in labor. The nurse assigned to care for the client ensures that which medication is readily accessible should respiratory depression occur?

--Naloxone Rationale: Opioid analgesics may be prescribed to relieve moderate to severe pain associated with labor. Opioid toxicity can occur and cause respiratory depression. Naloxone is an opioid antagonist, which reverses the effects of opioids and is given for respiratory depression. Morphine sulfate and hydromorphone hydrochloride are opioid analgesics. Betamethasone is a corticosteroid administered to enhance fetal lung maturity. (Saunders Maternal Meds)

Magnesium Sulfate (MgSO4) details:

--Toxicity: decrease patellar reflex, respirations depression, cardiac arrest, decreased urinary output. --Contraindications: Myasthenia gravis, renal failure --Antidote: Calcium gluconate - 1g over 5-10 minutes --Fetal effects: FHR baseline is lower and variability is decreased **Used prophylactically to prevent seizures **NOT a BP medication although it can lower BP

Severe complications of PIH / gestational hypertension include: ___

--placental separation: DIC; abruptio placentae --pulmonary edema --renal failure --stroke / CVA --IUGR [baby small for gestational age]; fetal death --HELLP syndrome: H - hemolysis (the breakdown of red blood cells; EL- elevated liver enzymes; LP - low platelet count.

The nurse has two priorities when the woman arrives at the birth center:

1. Establishing a Therapeutic Relationship 2. Assessing the condition of the mother and fetus

A client in labor is transported to the delivery room and prepared for a cesarean delivery. After the client is transferred to the delivery room table, the nurse should place the client in which position? 1. Supine position with a wedge under the right hip 2. Trendelenburg's position with the legs in stirrups 3. Prone position with the legs separated and elevated 4. Semi Fowler's position with a pillow under the knees

1. Supine position with a wedge under the right hip **Vena cava and descending aorta compression by the pregnant uterus impedes blood return from the lower trunk and extremities. This leads to decreasing cardiac return, cardiac output, and blood flow to the uterus and subsequently the fetus. The best position to prevent this would be side-lying, with the uterus displaced off the abdominal vessels. Positioning for abdominal surgery necessitates a supine position, however; a wedge placed under the right hip provides displacement of the uterus. Trendelenburg's position places pressure from the pregnant uterus on the diaphragm and lungs, decreasing respiratory capacity and oxygenation. A prone or semi Fowler's position is not practical for this type of abdominal surgery.

The nurse is caring for a client in labor. Which assessment findings indicate to the nurse that the client is beginning the second stage of labor? Select all that apply. 1. The contractions are regular. 2. The membranes have ruptured. 3. The cervix is dilated completely. 4. The client begins to expel clear vaginal fluid. 5. The spontaneous urge to push is initiated from the perineal pressure.

3 and 5 are correct - The second stage of labor begins with the cervix dilated completely and ends with the birth of the neonate. - The woman has a strong urge to push in stage 2 from the perineal pressure. **Test taking strategy -- answers 2 and 4 are comparable or alike; answer 1 begins before the 2nd stage. (Saunders 269-2)

___ = Paced breathing technique during which a woman breathes at approximately 32-40 breaths per minute (twice the normal breathing rate). Partner's involvement as birthing coach is stressed. Uses abdominal breathing techniques

Bradley

The mnemonic UNCOIL can help to remember nursing interventions to implement when the fetal heart monitor strip indicates late decelerations. (Lecture handout)

C-Change position of mother to her left side. O-Oxygen- Administer oxygen to mother to correct uteroplacental insufficiency. If OXYTOCIN is infusing ->STOP infusion which may be causing uterine hyperactivity resulting in uteroplacental insufficiency.. L-Lower head.- Lower the head of the bed to lower mother's head and elevate her feet to increase perfusion to uterus / fetus.

___ = Relaxed, deep breath in through the nose and out through the mouth that begins each breathing pattern and ends each contraction.

Cleansing breath

Regional anesthesia is used to anesthetize on REGION of the body; the client may remain awake throughout the procedure. What are some nursing care interventions related to this type of anesthesia? (Lecture handout)

Mnemonic: REGION R-Respiratory paralysis- Have ventilatory support equipment available. Avoid extreme Trendelenburg position before level of anesthesia is set. E-Elimination- Evaluate the bladder for distention. When the epidural is done on a pregnant woman, labor may be delayed due to bladder distention. G-Gastrointestinal- Check when the client last ate. Position to prevent aspiration. Antiemetics need to be available along with suction equipment. I-Inform of the procedure.- Does the client understand the procedure? Check for drug allergies. Make sure the legal consent form is signed. Have the client empty bladder prior to the procedure. O-Observe for hypotension-Report BP (blood pressure) of less than 100 systolic, or any significant decrease. Change the client's position, administer oxygen, and increase IV rate if the client is not prone to CHF. N-No trauma to extremities- Support extremities during movement. Remove legs from stirrups together.

The nurse is monitoring a client in active labor and notes that the client is having contractions every 3 minutes that last 45 seconds. The nurse notes that the fetal heart rate between contractions is 100 beats/minute. Which nursing action is most appropriate? 1. Notify the health care provider (HCP). 2. Continue monitoring the fetal heart rate. 3. Encourage the client to continue pushing with each contraction. 4. Instruct the client's coach to continue to encourage breathing techniques.

Notify the health care provider (HCP). Rationale: A normal fetal heart rate is 110 to 160 beats/minute, and the fetal heart rate should be within this range between contractions. Fetal bradycardia between contractions may indicate the need for immediate medical management, and the HCP or nurse-midwife needs to be notified.

Pharmacological pain management for labor:

Pharmacologic Methods **Analgesia: fentanyl, morphine, butorphinp; (Stadol), nalbuphine (Nubain) **Naloxone (Narcan) - narcotic antagonist **Tranquilizers Narcotic Potentiators: hydroxyzine (Vistaril) promethazine (Phenergan)

After the client vaginally delivers a viable newborn, the nurse sees the umbilical cord lengthen and observes a spurt of blood from the vagina. The nurse recognizes these findings as signs of which condition?

Placental separation

The nurse is administering an intravenous analgesic to a laboring woman. The woman inquires as to why the nurse is waiting for a contraction to begin before she infuses the medication into the intravenous line. Which is the nurse's most appropriate response?

**"Because the uterine blood vessels constrict during a contraction, the fetus will be less affected by the medication." **Rationale: Intravenous medication should be administered slowly in small doses starting at the beginning of a contraction and carrying over for 3 to 5 contractions. This intervention minimizes the amount of the medication that crosses the placenta and enters the fetal circulation, thus minimizing its effects on the fetus. Although this method of administration may decrease the amount of medication reaching the fetus, it does not totally eliminate effects of the medication on the fetus. (S-59)

The nurse is creating a plan of care for a pregnant client with a diagnosis of severe preeclampsia. Which nursing actions should be included in the care plan for this client? Select all that apply. 1. Keep the room semi-dark. 2. Initiate seizure precautions. 3. Pad the side rails of the bed. 4. Avoid environmental stimulation. 5. Allow out-of-bed activity as tolerated.

**1. Keep the room semi-dark. 2. Initiate seizure precautions. 3. Pad the side rails of the bed. 4. Avoid environmental stimulation. **Rationale: Clients with severe preeclampsia are maintained on bed rest in the lateral position. Only bathroom privileges may be allowed. Keeping the room semi-dark, initiating seizure precautions, and padding the side rails of the bed are accurate interventions. In addition, environmental stimuli such as interactions with visitors are kept at a minimum to avoid stimulating the client's central nervous system and causing a seizure. (S-42)

A client in labor is receiving oxytocin by intravenous infusion to stimulate uterine contractions. Which finding indicates that the rate of infusion needs to be decreased?

**A fetal heart rate of 180 beats/min **Rationale: A normal fetal heart rate is 110 to 160 beats/min. Acute hypoxia is a common cause of fetal tachycardia. The dosage of oxytocin should be decreased in the presence of fetal tachycardia, which can occur from excessive uterine activity. The goal of labor augmentation is to achieve 3 good-quality contractions (appropriate intensity and duration) in a 10-minute period. The uterus should return to resting tone between contractions, and there should be no evidence of fetal distress. Increased urinary output is unrelated to the use of oxytocin. (S-67)

The nurse is caring for a client in the active stage of labor. The nurse notes that the fetal pattern shows a late deceleration on the monitor strip. Based on this finding, the nurse should prepare for which appropriate nursing action?

**Administering oxygen via face mask **Rationale: Late decelerations are caused by uteroplacental insufficiency as a result of decreased blood flow and oxygen to the fetus during the uterine contractions. This causes hypoxemia; therefore, oxygen is necessary. The supine position is avoided because it decreases uterine blood flow to the fetus. The client should be turned on her side to displace pressure of the gravid uterus on the inferior vena cava. An IV oxytocin infusion is discontinued when a late deceleration is noted; otherwise, the oxytocin would cause further hypoxemia because of increased uteroplacental insufficiency caused by stimulation of contractions caused by the oxytocin. Documenting and monitoring would delay necessary treatment. (S-66)

The nurse is administering magnesium sulfate to a client for preeclampsia at 34 weeks' gestation. What is the priority nursing action for this client?

**Assess for signs and symptoms of labor. **Rationale: As a result of the sedative effect of the magnesium sulfate, the client may not perceive labor. (S-48)

The nurse assists the health care provider to perform an amniotomy on a client in labor. Which is the *priority* nursing action after this procedure?

**Assess the fetal heart rate. **Rationale: After amniotomy or rupture of the membranes in the birth setting, the nurse immediately assesses the fetal heart rate for at least 1 minute to detect changes associated with prolapse or compression of the umbilical cord. --The quantity, color, and odor of the amniotic fluid also are noted. The client's temperature should be assessed every 2 to 4 hours, and the nurse also should check the client's vital signs. The pads under the client should be changed regularly to promote comfort and reduce the moist environment that favors bacterial growth, but this is not the priority. (S-44)

After the spontaneous rupture of a laboring woman's membranes, the fetal heart rate drops to 85 beats/minute. Which should be the nurse's priority action?

**Assess the vagina and cervix with a gloved hand. **Rationale: It is most common to see an umbilical cord prolapsed directly after the rupture of membranes, when gravity washes the cord in front of the presenting part. A cord prolapse can be evidenced by fetal bradycardia with variable decelerations occurring with uterine contractions. Because the fetal heart rate became bradycardic immediately following the spontaneous rupture of the client's membranes, the nurse's initial action should be to glove the examining hand and insert 2 fingers into the vagina to assess for the presence of a prolapsed cord and then to relieve compression of the cord by exerting upward pressure on the presenting part. --Repositioning the woman to a knee-chest position is a correct intervention for prolapsed cord, but confirmation of the prolapsed cord and relieving compression is the first intervention that should be implemented. --An amnioinfusion may be used to minimize the effects of cord compression in utero, not a prolapsed cord(S-56)

The nurse is preparing to administer an analgesic to a client in labor. Which analgesic is contraindicated for a client who has a history of opioid dependency?

**Butorphanol tartrate ** Rationale: Butorphanol tartrate is an opioid analgesic that can precipitate withdrawal symptoms in an opioid-dependent client. Therefore, it is contraindicated if the client has a history of opioid dependency. -- Fentanyl, morphine sulfate, and meperidine are opioid analgesics but do not tend to precipitate withdrawal symptoms in opioid-dependent clients. (S-49)

A pregnant 39-week-gestation gravida 1, para 0 client arrives on the labor and delivery unit with signs and symptoms of active labor. The nurse reviews the client's prenatal record and discovers that she has had a positive group B streptococcus (GBS) laboratory report during her prenatal course. After performing a cervical exam, the nurse confirms that the cervix is dilated 6 cm and 90% effaced. Which should be the nurse's *first* action?

**Call the health care provider (HCP) to obtain a prescription for intravenous antibiotic prophylaxis (IAP). **Rationale: The client evidences progression toward delivery because the cervix is dilated 6 cm and the signs and symptoms of active labor are present. Because the client has had a positive GBS result during pregnancy, her neonate is at risk for becoming infected with GBS via vertical transmission during birth. GBS poses a significant risk for infant morbidity and mortality. To decrease this risk, it is recommended that IAP be administered during labor. (S-53)

The nurse is assisting in the care of a client in labor who is having an amniotomy performed. The nurse should report which abnormal findings to the health care provider (HCP)?

**Clear, dark amber amniotic fluid, Light green amniotic fluid with no odor, or Thick white amniotic fluid with no odor. **Rationale: Amniotic fluid is normally a pale straw color and may contain flecks of vernix caseosa. It should have a thin, watery consistency and may have a mild odor. The normal amount of amniotic fluid ranges from 500 to 1000 mL. Dark amber color, light green color, and thick white color are not descriptions of normal amniotic fluid and should be brought to the HCP's attention. (S-71)

The nurse in a delivery room is assessing a client immediately after delivery of the placenta. Which maternal observation could indicate uterine inversion and require immediate intervention?

**Complaints of severe abdominal pain **Rationale: Signs of uterine inversion include a depression in the fundal area, visualization of the interior of the uterus through the cervix or vagina, severe abdominal pain, hemorrhage, and shock. Chest pain and a rigid abdomen are signs of a ruptured uterus. A soft and boggy uterus indicates that the muscle is not contracting. (S-50)

The nurse has collected the following data on a client in labor. The fetal heart rate (FHR) is 154 beats/min and is regular, and contractions have moderate intensity, occur every 5 minutes, and have a duration of 35 seconds. Using this information, what is the appropriate action for the nurse to take?

**Continue to monitor the client. **Rationale: The data collected by the nurse are within normal limits and require no further action on the part of the nurse other than continued monitoring. The FHR is normally 110 to 160 beats/min. Signs of potential complications of labor include contractions consistently lasting 90 seconds or longer; contractions consistently occurring 2 minutes or less apart; fetal bradycardia, tachycardia, or persistently decreased variability; and irregular FHR. (S-79)

A labor room nurse is performing an assessment on a client in labor and notes that the fetal heart rate (FHR) is 158 beats/minute and regular. The client's contractions are every 5 minutes, with a duration of 40 seconds and of moderate intensity. On the basis of these assessment findings, what is the appropriate nursing action?

**Continue to monitor the client. **Rationale: The FHR normally is 110 to 160 beats/minute. Signs of potential complications of labor are contractions consistently lasting 90 seconds or longer or consistently occurring 2 minutes or less apart; fetal bradycardia, tachycardia, or persistently decreased variability; and irregular FHR. The assessment findings identified in the question are not signs of potential complications. (S-41)

The nurse is preparing to care for a client in labor. The health care provider (HCP) has prescribed an intravenous (IV) infusion of oxytocin. The nurse should ensure that which is implemented before the beginning of the infusion?

**Continuous electronic fetal monitoring **Rationale: Continuous electronic fetal monitoring should be implemented during an IV infusion of oxytocin. (S-70)

The nurse is preparing to care for a client in labor. The health care provider has prescribed an intravenous (IV) infusion of oxytocin. The nurse ensures that which intervention is implemented before initiating the infusion?

**Continuous electronic fetal monitoring **Rationale: Oxytocin is a uterine stimulant used to induce labor. Continuous electronic fetal monitoring should be implemented during an IV infusion of oxytocin. (S-32)

An ultrasound is performed on a client with suspected abruptio placentae, and the results indicate that a placental abruption is present. Which intervention should the nurse prepare the client for?

**Delivery of the fetus **Rationale: The goal of management in abruptio placentae is to control the hemorrhage and deliver the fetus as soon as possible. (S-77)

The nurse is monitoring a client in labor whose membranes ruptured spontaneously. What is the initial nursing action?

**Determine the fetal heart rate. **Rationale: When the membranes rupture in the birth setting, the nurse immediately assesses the fetal heart rate to detect changes associated with prolapse or compression of the umbilical cord. Taking the blood pressure and noting the characteristics of the amniotic fluid are also appropriate actions, but are not the initial actions in this situation. The nurse may assist the client in cleaning, changing clothing, and providing peripads, but determining the fetal heart rate is the initial action. (S-68)

The nurse in the labor room is caring for a client who is in the first stage of labor. On assessing the fetal patterns, the nurse notes an early deceleration of the fetal heart rate (FHR) on the monitor strip. Based on this finding, which is the appropriate nursing action?

**Document the findings and continue to monitor fetal patterns. **Rationale: Early deceleration of the FHR refers to a gradual decrease in the heart rate, followed by a return to baseline, in response to compression of the fetal head. It is a normal and benign finding. Because early decelerations are considered benign, interventions are not necessary. (S-63)

The nurse is caring for a client in active labor. Which nursing intervention would be the best method to prevent fetal heart rate (FHR) decelerations?

**Encourage an upright or side-lying maternal position. **Rationale: Side-lying and upright positions such as walking, standing, and squatting can improve venous return and encourage effective uterine activity. Many nursing actions are available to prevent FHR decelerations, without necessitating surgical intervention. The nurse should discontinue an oxytocin infusion in the presence of FHR decelerations, thereby reducing uterine activity and increasing uteroplacental perfusion.(S-47)

A pregnant 39-week-gestation client arrives at the labor and delivery unit in active labor. On confirmation of labor, the client reports a history of herpes simplex virus (HSV) to the nurse, who notes the "presence of lesions" on inspection of the client's perineum. Which should be the nurse's initial action?

**Explain to the client why a cesarean delivery is necessary. **Rationale: Because neonatal infection of HSV is life-threatening, prevention of neonatal infection is critical. Current recommendations state that a cesarean delivery within 4 hours after labor begins or membranes rupture is necessary if visible lesions are present on the woman's perineum. --At this phase in the client's pregnancy, the focus is on preventing transmission to the fetus rather than controlling the symptoms of HSV. (S-54)

An initial focused assessment is done before the broader database assessment to determine the condition of the mother and fetus to determine whether birth is imminent. What is typically included in the focused assessment?

**FETAL ASSESSMENT Review prenatal record for gestational age Assess FHR, fetal movement, membrane status **MATERNAL ASSESSMENT Vital Signs Assess for signs of hypertension or infection Uterine contraction pattern **IMPENDING BIRTH Grunting sounds Sitting on buttocks Saying "something is coming" *NOTE: If focused assessment of mother and fetus are normal and birth is not imminent, complete the admission assessment. If the admission assessment are not normal or birth is near, notify physician or nurse-midwife promptly*

Parameters of stages of labor:

**First stage: begins with onset of U/C and ends at 10 cms --Latent Phase = 0-3 cms 100% -2, Tip of nose, P-0 =6hrs, > P-1 = 4/5 hrs U/C's mild, q5-30 min, 20-40 secs --Active Phase = 3cms-7cms, 0, Chin, P-0 = 3, >P-1 = 2, Analgesics do not affect labor at this time, Show, SROM, U/C's q 3-5 min, 40-60, --Transition = 8cms-10, +1-+2, Forehead, Intense pain, N&V, irritability, anxiety, panic, loss of control **Second stage- Begins at 10cms and ends with delivery= Expulsion of baby P-0 = pushing effort up to 3 hrs, >P-1= 30-60 min or imminent delivery with minimal pushing effort, U/C's 3-4 min, 60-90, uncontrollable urge to push or bear down, Preparing for birth -Maternal position, Bearing-down efforts FHR and pattern, Support of father or partner, Supplies, instruments, and equipment, ***Mechanism of birth: vertex presentation, Birth of head, Birth of shoulders, Birth of body and extremities **Third stage -Begins with delivery of neonate and ends with delivery of placenta, Within 30 min **Fourth stage-Begins with placental delivery, 1-4 hrs, Initiates pp period, Risk for hemorrhage, bladder distention, DVT

The nurse is monitoring a client who is in the active phase of labor. The client has been experiencing contractions that are short, irregular, and weak. Which type of labor dystocia should the nurse document that the client is experiencing?

**Hypotonic **Rationale: Hypotonic labor contractions are short, irregular, and weak and usually occur during the active phase of labor. Precipitate labor is that which lasts in its entirety for 3 hours or less. Hypertonic dysfunction usually occurs during the latent phase of labor. Preterm labor is the onset of labor after 20 weeks of gestation and before the beginning of the 38th week of gestation. (S-78)

Nursing care after epidural procedure:

**Monitor maternal vital signs & FHR q 5 min initially and after every re-bolus, then every 15 mins and manage hypotension or alteration in FHR **Manage elimination- catheterization may be required **Assess pain and level of sensation and motor loss **Position woman as needed (on side to prevent inferior vena cava syndrome) **Have O2 & Ephedrine ( a vasopressor) available to treat hypotension **Maintain continuous infusion as ordered **Apply warm blankets due vasodilation **Document all pain characteristics, including location, intensity, quality, frequency, duration and effectiveness of relief measures

The nurse is creating a plan of care for a client experiencing dystocia and includes several nursing interventions in the plan. The nurse prioritizes the plan and selects which nursing intervention as the highest priority?

**Monitoring fetal status **Rationale: The priority in the plan of care should include the intervention that addresses the physiological integrity of the fetus. Although providing comfort measures, changing the client's position frequently, and keeping the significant other informed of the progress of the labor are components of the plan of care, fetal status is the priority. (S-72)

Nursing care prior to epidural procedure:

**Obtain consent(Informed Consent per anesthesia) **Check lab values (PT, PTT & Platelet count)- for bleeding and clotting abnormalities **IV fluids- 500-1000cc bolus of LR or NS **Have Ephedrine readily available for hypotension **Assist with mother positioning per anesthesia **Ensure emergency equipment is available Do time-out procedure ( Joint Commission- Safety) See Nursing Process: Discomfort in Labor, p. 417 See Emergency: Maternal Hypotension w/Decreased Placenta Perfusion

An amniotomy is performed on a client in labor. On the amniotic fluid examination, the delivery room nurse should identify which findings as normal?

**Pale straw in color, with flecks of vernix **Rationale: Amniotic fluid normally is pale straw in color and may contain flecks of vernix caseosa. --Greenish fluid may indicate the presence of meconium and suggests fetal distress. --Amber-colored fluid suggests the presence of bilirubin. --The fluid should not be thick and white; this could be an indication of infection.(S-40)

A woman in active labor has requested a regional anesthetic. She is currently 5 cm dilated. The health care provider has prescribed an epidural block. Which nursing intervention should be implemented after the epidural block has been placed?

**Palpate the bladder at frequent intervals. **Rationale: The effect of the epidural is that anesthesia is felt from the fifth lumbar space to the sacral region of the vertebral column. The woman loses the sensation that she needs to urinate. The nurse must palpate the bladder frequently because a full bladder will impede progression of the fetus during the laboring process. Ambulation is not allowed because of the anesthesia. The woman is encouraged to lie on her side to increase placental perfusion to the fetus. Hypotension, not hypertension, is a concern. (S-62)

Labor anesthesia notes:

**Paracervical used during the first stage of labor to relieve pain from uterine contractions and cervical dilation, rarely used for labor because of its association with fetal bradycardia. **Epidural- the most effective pharmacological pain relief method currently available. For relieving the discomfort of labor and vaginal birth, a block from T10 to S5 is required **Spinal- not suitable for labor, used for C/section, provides anesthesia from nipple (T6 to feet) **Pudendal- administered late in second stage, useful for an episiotomy or if forceps or vacuum extractor is used to facilitate birth.

The nurse performs a vaginal assessment on a pregnant client in labor. On assessment, the nurse notes the presence of the umbilical cord protruding from the vagina. Which is the initial nursing action?

**Place the client in Trendelenburg's position. **Rationale: When cord prolapse occurs, prompt actions are taken to relieve cord compression and increase fetal oxygenation. The mother should be positioned with her hips higher than her head to shift the fetal presenting part toward the diaphragm. The nurse should push the call light to summon help, and other staff members should call the HCP and notify the delivery room. If the cord is protruding from the vagina, no attempt should be made to replace it because that could traumatize it and further reduce blood flow. Oxygen at 8 to 10 L/min by face mask is administered to the mother to increase fetal oxygenation. (S-75)

The nurse assists in the vaginal delivery of a newborn. Following the delivery, the nurse observes the umbilical cord lengthen and a spurt of blood from the vagina. The nurse should document these observations as signs of which condition?

**Placental separation **Rationale: As the placenta separates, it settles downward into the lower uterine segment. The umbilical cord lengthens, and a sudden trickle or spurt of blood appears. (S-69)

The nurse is preparing to care for a client with hypertonic labor. The nurse is told that the client is experiencing uncoordinated contractions that are erratic in their frequency, duration, and intensity. Which is the priority nursing intervention?

**Provide pain relief measures. **Rationale: Management of hypertonic labor depends on the cause. Relief of pain is the primary intervention to promote a normal labor pattern. Therapeutic management for hypotonic uterine dysfunction includes amniotomy and oxytocin augmentation to stimulate a labor that slows. The client with hypertonic uterine dysfunction should not be encouraged to ambulate every 30 minutes but should be encouraged to rest. (S-74)

The nurse is assessing the deep tendon reflexes of a client with severe preeclampsia who is receiving intravenous magnesium sulfate. The nurse should perform which procedure to assess the brachioradialis reflex? Click on the image to indicate your answer. (From Jarvis [2013], pp. 646-648.)

**Rationale: To assess the brachioradialis reflex, the client's thumb is held to suspend the forearm in relaxation. The nurse then strikes the forearm directly, about 2 to 3 cm above the radial styloid process. The normal response is flexion and supination of the forearm. (S-46)

The purpose of a vaginal examination for a client in labor is to specifically assess the status of which findings?

**Station, Dilation, Effacement **Rationale: The vaginal examination for a client in labor specifically determines effacement 0% to 100%, dilation 0 to 10 cm, and station -5 cm (above the maternal ischial spine) to +5 cm (below the maternal ischial spine). Bloody show is the brownish or blood-tinged cervical mucus that may be passed preceding labor and is not a specific part of the assessment when performing a vaginal examination. Contraction effort is not determined by vaginal examination. (S-81)

The nurse is caring for a client who is receiving oxytocin for induction of labor and notes a nonreassuring fetal heart rate (FHR) pattern on the fetal monitor. On the basis of this finding, the nurse should take which action *first*?

**Stop the oxytocin infusion. **Rationale: Oxytocin stimulates uterine contractions and is used to induce labor. If uterine hypertonicity or a nonreassuring FHR pattern occurs, the nurse needs to intervene to reduce uterine activity and increase fetal oxygenation. The oxytocin infusion is stopped, the client is placed in a side-lying position, and oxygen by face mask at 8 to 10 L/min is administered. The health care provider is notified. The nurse should monitor the client's blood pressure and intake and output; however, the nurse should first stop the infusion. (S-64)

The nurse is caring for a client in labor and notes that minimal variability is present on a fetal heart rate (FHR) monitor strip. Which conditions are most likely associated with minimal variability?

**Tachycardia, Fetal hypoxia, Metabolic acidemia, Congenital anomalies **Rationale: The fluctuations in the baseline FHR are the definition of variability. Variability can be classified into 4 different categories: absent, minimal, moderate, and marked. Minimal variability is defined as fluctuations that are fewer than 6 beats/minute. Tachycardia, fetal hypoxia, metabolic acidemia, and congenital anomalies are all associated with possible minimal variability. Rupturing membranes and early labor are not correlated to this condition. (S-55)

On March 10, the nurse performed an initial assessment on a client admitted to the labor and delivery unit for "rule out labor." The client has not received prenatal care but is certain that the first day of her last menstrual period (LMP) was July 7 the previous year. The nurse plans care based on which interpretation?

**The client is possibly in preterm labor. **Rationale: According to Nägele's rule, by subtracting 3 months and adding 7 days and 1 year to this client's LMP the nurse can determine that her estimated date of delivery (EDD) is April 14. This client is in the labor and delivery unit to be evaluated for the presence of labor more than 1 month before her EDD; therefore, she is possibly in preterm labor. Viability is said to occur between the 22nd and 25th weeks of gestation. This fetus is approximately 4 weeks before term. If this client truly is in labor, the health care provider's plan would be to try to stop the labor in order to prevent delivery at this early stage in the pregnancy. (S-60)

The nurse is caring for a client during the second stage of labor. On assessment, the nurse notes a slowing of the fetal heart rate and a loss of variability. What is the initial nursing action?

**Turn the client on her side and administer oxygen by face mask at 8 to 10 L/min. **Rationale: If a fetal heart rate begins to slow or a loss of variability is observed, this could indicate fetal distress. To facilitate oxygen to the mother and her fetus, the client is turned to her side, which reduces the pressure of the uterus on the ascending vena cava and descending aorta. Oxygen at 8 to 10 L/min is applied to the mother by face mask. (S-76)

The nurse is caring for a client during the second stage of labor. On assessment, the nurse notes a slowing of the fetal heart rate and a loss of variability. Which is the initial nursing action?

**Turn the client onto her side and give oxygen by face mask at 8 to 10 L/min. **Rationale: If a fetal heart rate begins to slow or a loss of variability is observed, this could indicate fetal distress. To promote adequate oxygenation for the mother and her fetus, the mother is turned onto her side, which reduces the pressure of the uterus on the ascending vena cava and descending aorta. Oxygen by face mask at 8 to 10 L/min is then applied to the mother.(S-39)

Shortly after receiving epidural anesthesia, a laboring woman's blood pressure drops to 95/43 mm Hg. Which immediate actions should the nurse take?

**Turn the woman to a lateral position, Increase the rate of the intravenous infusion, and Administer oxygen by face mask at 10 L/minute. **Rationale: Maternal hypotension results in decreased placental perfusion, so the focus of nursing care should be to initiate interventions that increase oxygen perfusion to the fetus. Turning the woman to left lateral position assists in deflecting the uterus off of the vena cava, thus improving maternal circulation. Increasing the rate of the intravenous infusion will increase blood volume, which will increase the maternal blood pressure. An increase in blood pressure would increase placental perfusion. Administering a high flow rate of oxygen will increase the oxygen levels in the maternal circulation and increase oxygen delivery to the fetus. (S-58)

The nurse is assessing a pregnant client in the second trimester of pregnancy who was admitted to the maternity unit with a suspected diagnosis of abruptio placentae. Which assessment finding should the nurse expect to note if this condition is present?

**Uterine tenderness **Rationale: Abruptio placentae is the premature separation of the placenta from the uterine wall after the 20th week of gestation and before the fetus is delivered. In abruptio placentae, acute abdominal pain is present. Uterine tenderness accompanies placental abruption, especially with a central abruption and trapped blood behind the placenta. The abdomen feels hard and boardlike on palpation as the blood penetrates the myometrium and causes uterine irritability. --A soft abdomen and painless, bright red vaginal bleeding in the second or third trimester of pregnancy are signs of placenta previa. (S-16)

The nurse is collecting data from a pregnant client in the second trimester of pregnancy who was admitted to the maternity unit with a suspected diagnosis of abruptio placentae. Which findings are associated with abruptio placentae?

**Uterine tenderness, Acute abdominal pain, A hard, "boardlike" abdomen **Rationale: In abruptio placentae, acute abdominal pain is present. Uterine tenderness accompanies placental abruption, especially with a central abruption and trapped blood behind the placenta. The abdomen will feel hard and boardlike on palpation as the blood penetrates the myometrium and causes uterine irritability. Observation of the fetal monitoring often reveals increased uterine resting tone, caused by placental abruption. Painless, bright red vaginal bleeding in the second or third trimester of pregnancy is a sign of placenta previa. (S-82)

A pregnant client admitted to the labor room arrived with a fetal heart rate (FHR) of 94 beats/minute and the umbilical cord protruding from the vagina. The client tells the nurse that her "water broke" before coming to the hospital. What is the appropriate nursing action?

**Wrap the cord loosely in a sterile towel soaked with warm, sterile normal saline. **Rationale: When an umbilical cord is protruding, the cord must be protected from drying out and becoming compressed. Wrapping the cord with a sterile, saline-soaked towel will help accomplish this. The nurse must also help reduce compression of the cord by placing the client in an extreme Trendelenburg's or modified Sims' position. The health care provider is also notified immediately. A tocolytic would be used if the client had inadequate uterine relaxation. IV solutions may be administered but are not the priority item with the information given. (S-80)

What is EMTALA (Emergency Medical Treatment and Active Labor Act)?

*Federal regulation *Ensures that women gets emergency treatment or active labor care -True labor -Decreased fetal movement -Rupture of membranes *The nurse must document -Assessment findings -Interventions implemented -Client responses to care measures

Maternal hypertension may have what effect on pregnancy? (Lecture / powerpoint/ PIH)

*Hypertension associated with uterine growth blood vessels narrowing result in decreased oxygen and nutrients to the fetus resulting in IUGR

What are the signs and symptoms of preeclampsia? (Lecture / powerpoint/ PIH)

*MILD PREECLAMPSIA: -High blood pressure: 140/90 -Proteinuria 1+ to 2+ -Edema: lower legs and feet: 1+ to 2+ - less than 1 lb/week gain Reflexes: 1+ to 2+ brisk **SEVERE PREECLAMPSIA -High blood pressure: 160/110 -Proteinuria 3+ to 4+ -Edema: -Reflexes: 3+ to 4+ hyperreflexia / clonus -Blurred vision / retinal detachment/ Scotoma -Nausea/vomiting; RUQ / epigastric pain -Headache / LOC changes due to decreased cerebral perfusion -Heart: decreased cardiac output leading to pulmonary edema -Shortness of breath, pulmonary edema, coughing up blood. -Changes in liver enzymes; severe heart burn -Premature aging of placenta; IUGR, late decelerations: INCREASED RISK placenta abruption / fetal death.

Side effects of systemic analgesia / misc. notes:

*Naloxone used to reverse the side effects of narcotic opiods *Potentiator- increases the effects of the narcotic *Stadol and, nubain- opiod with mixed agonist-antagonist effects (stimulate opioid recepter ->analgesia), drowsiness, dizziness, lightheadedness, euphoria, nausea, clammy skin, sweating, insomnia, abdominal pain, constipation, resp depression, shock-decrease maternal pulse, EFM decrease variability- *Nubain onset 2-3 minutes IV, half-life approx 5 hours- Stadol- a few minutes onset IV, peak 30-60 mins, half-life approx 3-4 hrs *Phenergan- antiemetic, antihistaminic, sedative, anticholinergic, anti-motion sickness- onset 5 min, half life 9-16 hrs, potentiates opioids, no antagonist *Narcan- opioid antagonist- prevent the effects of opioids- muscle and joint pains, difficulty sleeping, anxiety, HA, nervousness, vomiting, abd pain, hepatotoxicity- used to reverse resp depression, sedation, hypotension- onset 2 minutes IV- may have shorter half-life than opioids so s/s opioid toxicity may return- *Oxytocin- destroyed by gut therefore has to be IV or IM-myometrium has receptor sites which influence strength of u/c, half life 1-6 minutes, HTN, antidiuretic properties- uterine rupture, water intoxication, fetal bradycardia- other ocytocics= misoprostol- uterine cramping, n&v, diarrhea, ha, uterine rupture

Typical elements of focused fetal assessment:

*Obtain a 20-30 fetal monitoring baseline strip (>24 wks) *Auscultate with doppler for IUP<24 weeks *Basic information (reason for coming) *Prenatal care information from ACOG form *Labor status *Physical examination (brief if imminent delivery) to determine overall health

What is fetal scalp blood sampling? (Lecture / powerpoint)

*Power point note: requires rupture of membranes and acidosis is present if pH is less than 7.20 [Google: Fetal scalp blood testing is a technique used in obstetrics during labor to confirm whether fetal oxygenation is sufficient. The procedure can be performed by creating a shallow cut by a transvaginally inserted blood lancet, followed by applying a thin pipe to the site that samples blood by capillary action.]

What is proteinuria?

*Proteinuria is one of the cardinal features of preeclampsia, a common and potentially severe complication of pregnancy. *Proteinuria in pregnancy can also indicate primary renal disease or renal disease secondary to systemic disorders, such as diabetes or primary hypertension.

What are predisposing factors for preeclampsia? (Lecture / powerpoint/ PIH)

*primagravida under 17 or over 35 *multiple pregnancy *obesity *chronic hypertension *vascular disease: diabetes, renal *family history *hydatiform mole *lower socioeconomic status: severe malnutrition, decreased protein intake, inadequate or late prenatal care

Butorphanol tartrate by intravenous push is prescribed for a client in labor. The nurse recognizes which assessment findings to be side or adverse effects of this medication?

-- Tinnitus, Syncope, Palpitations, Nausea and vomiting Rationale: The client in labor may be given parenteral analgesia during the first stage of labor, up to 2 to 3 hours before the anticipated delivery. Butorphanol tartrate is a medication that may be prescribed for pain relief. (Saunders Maternal Meds)

The senior nursing student is assigned to care for a client with severe preeclampsia who is receiving an intravenous infusion of magnesium sulfate. The co-assigned registered nurse asks the student to describe the actions and effects of this medication. Which statement, if made by the student, indicates the need for further teaching?

--"It increases acetylcholine, blocking neuromuscular transmission." Rationale: Magnesium sulfate produces flushing and sweating because of decreased peripheral blood pressure. It decreases the frequency and duration of uterine contractions and decreases central nervous system activity, acting as an anticonvulsant. Magnesium sulfate decreases (not increases) acetylcholine, blocking neuromuscular transmission. (Saunders Maternal Meds)

Butorphanol tartrate is prescribed for a woman in labor, and the woman asks the nurse about the purpose of the medication. The nurse should make which appropriate response?

--"The medication provides pain relief during labor." Rationale: Butorphanol tartrate is an opioid analgesic that provides systemic pain relief during labor. (Saunders Maternal Meds)

A client diagnosed with severe preeclampsia is receiving magnesium sulfate by continuous intravenous infusion. Which assessment finding would indicate that the medication should be discontinued?

--Absence of deep tendon reflexes Rationale: Signs of magnesium toxicity include central nervous system depression. The respiratory system will fail with the absence of deep tendon reflexes if this condition is not corrected. The client should maintain a respiratory rate at or greater than 16 breaths per minute (or per agency protocol), maintain the presence of deep tendon reflexes, and maintain a urinary output greater than 30 mL/hour. A decrease in blood pressure is a positive finding because preeclampsia is accompanied by hypertension. (Saunders Maternal Meds)

Rho(D) immune globulin is prescribed for a client after delivery and the nurse provides information to the client about the purpose of the medication. The nurse determines that the woman understands the purpose if the woman states that it will protect her next baby from which condition?

--Being affected by Rh incompatibility Rationale: Rh incompatibility can occur when an Rh-negative mother becomes sensitized to the Rh antigen. Sensitization may develop when an Rh-negative woman becomes pregnant with a fetus who is Rh positive. During pregnancy and at delivery, some of the fetus's Rh-positive blood can enter the maternal circulation, causing the mother's immune system to form antibodies against Rh-positive blood. Administration of Rho(D) immune globulin prevents the mother from developing antibodies against Rh-positive blood by providing passive antibody protection against the Rh antigen. (Saunders Maternal Meds)

A client in preterm labor (31 weeks) who is dilated to 4 cm has been started on magnesium sulfate and contractions have stopped. If the client's labor can be inhibited for the next 48 hours, the nurse anticipates a prescription for which medication?

--Betamethasone **Rationale: Betamethasone, a glucocorticoid, is given to increase the production of surfactant to stimulate fetal lung maturation. It is administered to clients in preterm labor at 28 to 32 weeks of gestation if the labor can be inhibited for 48 hours. *Notes: Nalbuphine is an opioid analgesic. Rho(D) immune globulin is given to Rh-negative clients to prevent sensitization. Dinoprostone vaginal insert is a prostaglandin given to ripen and soften the cervix and to stimulate uterine contractions. (Saunders Maternal Meds)

Methylergonovine is prescribed for a woman to treat postpartum hemorrhage. Before administration of methylergonovine, what is the priority assessment?

--Blood Pressure Rationale: Methylergonovine, an ergot alkaloid, is used to prevent or control postpartum hemorrhage by contracting the uterus. Methylergonovine causes continuous uterine contractions and may elevate the blood pressure. A priority assessment before the administration of the medication is to check the blood pressure. The health care provider needs to be notified if hypertension is present. (Saunders Maternal Meds)

Methylergonovine is prescribed for a woman with postpartum hemorrhage caused by uterine atony. Before administering the medication, the nurse should check which most important client parameter?

--Blood pressure Rationale: Methylergonovine is an ergot alkaloid used for postpartum hemorrhage. It stimulates contraction of the uterus and causes arterial vasoconstriction. Ergot alkaloids are avoided in women with significant cardiovascular disease, peripheral disease, hypertension, eclampsia, or preeclampsia. Such conditions are worsened by the vasoconstrictive effects of the ergot alkaloids. The nurse should assess the woman's blood pressure before administering the medication and should follow agency protocols regarding withholding of the medication. (Saunders Maternal Meds)

The nurse in the postpartum unit notes that a new mother was given methylergonovine intramuscularly following delivery. What assessment finding indicates that the medication was effective?

--Decreased uterine bleeding Rationale: Methylergonovine, an oxytocic, is an agent that is used to prevent or control postpartum hemorrhage by contracting the uterus. The immediate dose usually is given intramuscularly; if additional medication is needed, it is given by mouth. No relationship exists between the action of this medication and lochial drainage. This medication may elevate blood pressure and increase the strength and frequency of contractions. A priority assessment component before the administration of methylergonovine is blood pressure. (Saunders Maternal Meds)

On assessment, a newborn is exhibiting cyanosis, tachypnea, nasal flaring, and grunting. Respiratory distress syndrome is diagnosed, and the health care provider (HCP) prescribes surfactant replacement therapy. Through which route should the nurse prepare to administer this medication?

--Endotracheally through the endotracheal tube Rationale: Respiratory distress syndrome is a serious lung disorder caused by immaturity and inability to produce surfactant, resulting in hypoxia and acidosis. The aim of therapy in respiratory distress syndrome is to support the disease until it runs its course, with the subsequent development of surfactant. The infant may benefit from surfactant replacement therapy. In surfactant replacement, an exogenous surfactant preparation is instilled into the lungs through an endotracheal tube. (Saunders Maternal Meds)

A pregnant woman of 30 weeks' gestation is admitted to the maternity unit in preterm labor. The woman asks the nurse about the purpose of betamethasone, which has been prescribed by the health care provider (HCP). The nurse should tell the client that the medication will promote which action?

--Enhance fetal lung maturity. Rationale: Betamethasone, a steroidal antiinflammatory, increases the surfactant level and promotes lung maturation, thereby reducing the risk of respiratory distress syndrome in the newborn infant. Surfactant production does not become stable until after 32 weeks of gestation, and if adequate amounts of lung surfactant are not present, respiratory distress and death of the newborn infant could result. Delivery should be delayed for at least 48 hours after administration of betamethasone to allow time for the lungs to mature. (Saunders Maternal Meds)

The nurse is monitoring a client in preterm labor who is receiving intravenous magnesium sulfate. The nurse should monitor for which adverse effects of this medication?

--Flushing, Depressed respirations, Extreme muscle weakness Rationale: Magnesium sulfate is a central nervous system depressant and relaxes smooth muscle, including the uterus. It is used to halt preterm labor contractions and is used for preeclamptic clients to prevent seizures. Adverse effects include flushing, depressed respirations, depressed deep tendon reflexes, hypotension, extreme muscle weakness, decreased urine output, pulmonary edema, and elevated serum magnesium levels.(Saunders Maternal Meds)

Methylergonovine is prescribed for a client with postpartum hemorrhage. Before administering the medication, the nurse should contact the health care provider who prescribed the medication if which condition is documented in the client's medical history?

--Peripheral vascular disease Rationale: Methylergonovine is an ergot alkaloid used to treat postpartum hemorrhage. Ergot alkaloids are contraindicated in clients with significant cardiovascular disease, peripheral vascular disease, hypertension, preeclampsia, or eclampsia. These conditions are worsened by the vasoconstrictive effects of the ergot alkaloids. (Saunders Maternal Meds)

A pregnant client is receiving magnesium sulfate for the management of preeclampsia. The nurse determines that the client is experiencing toxicity from the medication if which findings are noted on assessment?

--Respirations of 10 breaths/minute --Urine output of 20 mL in an hour Rationale: Magnesium toxicity can occur from magnesium sulfate therapy. Signs of magnesium sulfate toxicity relate to the central nervous system depressant effects of the medication and include respiratory depression, loss of deep tendon reflexes, and a sudden decline in fetal heart rate and maternal heart rate and blood pressure. Respiratory rate below 12 breaths per minute is a sign of toxicity. Urine output should be at least 25 to 30 mL per hour. Proteinuria of 3+ is an expected finding in a client with preeclampsia. Presence of deep tendon reflexes is a normal and expected finding. Therapeutic serum levels of magnesium are 4 to 7.5 mEq/L (2 to 3.75 mmol/L). (Saunders Maternal Meds)

A client experiencing preterm labor at the 29th week of gestation has been admitted to the hospital. The client has a prescription to receive betamethasone but delivers too quickly for medication administration. As a result of not receiving this medication, which condition is most likely to develop in the preterm newborn?

--Respiratory depression Rationale: Betamethasone is classified as an antiinflammatory and corticosteroid. It increases the surfactant level and lung maturity in the fetus, which reduces the incidence of respiratory distress syndrome. Delivery must be delayed for at least 48 hours after administration of betamethasone to allow time for the lungs of the fetus to mature. (Saunders Maternal Meds)

A client with preeclampsia is receiving magnesium sulfate. The nurse should assess the client closely for which sign of magnesium toxicity?

--Respiratory rate of 10 breaths/minute Rationale: Magnesium toxicity is a risk associated with magnesium sulfate therapy. Signs of magnesium toxicity relate to central nervous system (CNS) depression and include respiratory depression, loss of deep tendon reflexes, and sudden drop in fetal heart rate and/or maternal heart rate and blood pressure. Magnesium is excreted through the kidneys. If renal impairment is present, magnesium toxicity can develop very quickly. Therapeutic serum levels of magnesium are 4 to 7 mEq/L (2 to 3.5 mmol/L). (Saunders Maternal Meds)

A woman with preeclampsia is receiving magnesium sulfate. Which indicates to the nurse that the magnesium sulfate therapy is effective?

--Seizures do not occur. Rationale: For a client with preeclampsia, the goal of care is directed at preventing eclampsia (seizures). Scotomas are areas of complete or partial blindness. Visual disturbances, such as scotomas, often precede an eclamptic seizure. Ankle clonus indicates hyperreflexia and may precede the onset of eclampsia. Magnesium sulfate is an anticonvulsant, not an antihypertensive agent. Although a decrease in blood pressure may be noted initially, this effect is usually transient. (Saunders Maternal Meds)

The nurse performs an assessment of a pregnant woman who is receiving intravenous magnesium sulfate for management of preeclampsia and notes that the woman's deep tendon reflexes are absent. On the basis of this finding, the nurse should make which interpretation?

--The woman is experiencing magnesium excess. Rationale: Magnesium toxicity can occur as a result of magnesium sulfate therapy. Signs of magnesium sulfate toxicity relate to the central nervous system depressant effects of the medication and include respiratory depression, loss of deep tendon reflexes, sudden decrease in fetal heart rate or maternal heart rate or both, and sudden drop in blood pressure. An absence of reflexes indicates magnesium excess and toxicity. The infusion rate therefore would not be increased. Hyperreflexia indicates increased cerebral edema.(Saunders Maternal Meds)

The nurse is monitoring a client who is receiving oxytocin to induce labor. Which assessment findings should cause the nurse to immediately discontinue the oxytocin infusion?

--Uterine hyperstimulation --Late decelerations of the fetal heart rate Rationale: Oxytocin stimulates uterine contractions and is a pharmacological method to induce labor. Late decelerations, a nonreassuring fetal heart rate pattern, is an ominous sign indicating fetal distress. Oxytocin infusion must be stopped when any signs of uterine hyperstimulation, late decelerations, or other adverse effects occur. Some health care providers prescribe the administration of oxytocin in 10-minute pulsed infusions rather than as a continuous infusion. This pulsed method, which is more like endogenous secretion of oxytocin, is reported to be effective for labor induction and requires significantly less oxytocin use. (Saunders Maternal Meds)

A pregnant client is receiving oxytocin for the induction of labor. The nurse should immediately discontinue the oxytocin infusion if which is noted in the client?

--Uterine hyperstimulation Rationale: Oxytocin is a synthetic hormone that stimulates uterine contractions and commonly is used to induce labor. A major danger associated with oxytocin induction of labor is hyperstimulation of uterine contractions, which can cause fetal distress as a result of decreased placental perfusion. Therefore, oxytocin infusion must be stopped when any signs of uterine hyperstimulation are observed. (Saunders Maternal Meds)

Nursing Interventions for management of MgSO4:

1. Fetal monitoring 2. Fluid management: [preeclampsia vasoconstriction with leaky blood vessels so fluid leaks into 3rd space or lungs] -- risk of pulmonary edema; expect some oliguria; diuretics only for edema; fluid restrictions; Manage BP; Anesthesia: risk of hypotension, hematoma with low platelet count, GETA-general endotracheal anesthesia can cause spike in BP and stroke, induction risk

A client arrives at a birthing center in active labor. Following examination, it is determined that her membranes are still intact and she is at -2 station. The HCP prepares to perform an amniotomy. What will the nurse relay to the client as the "most likely" outcomes of the amniotomy? Select all that apply. 1. Less pressure on her cervix 2. Decrease the number of contractions 3. Increased efficiency of contractions 4. The need for increased maternal blood pressure monitoring 5. The need for frequent FHR monitoring to detect the presence of a prolapsed cord.

3 and 5 are correct. 3. Increased efficiency of contractions. 5. The need for frequent FHR monitoring to detect the presence of a prolapsed cord. **Amniotomy -aka artificial rupture of membranes- can be used to induce labor when the cervix is favorable (ripe) or to augment labor if the progress begins to slow. Rupturing the membranes allows the fetal head to contact the cervix more directly and may increase the efficiency of contractions. (S-273-6)

The 3rd stage of labor: Placenta Separation

3rd stage of labor- 1. Placenta begins the separation process in central portion with retroplacental bleeding. Uterus changes from discoid to globular shape. 2. Placenta completes separation and enters the lower uterine segment. The uterus is globular in shape. 3. Placenta enters the vagina, the cord is seen to lengthen, and there may be increased bleeding. 4. Expulsion (birth) of placenta and completion of third stage

When to Go to the Hospital or Birth Center

CONTRACTIONS: A pattern of increasing regularity, frequency, duration and intensity **Nullipara: regular contractions 5 minutes ,apart, for 1 hour **Multipara: regular contractions, 10 minutes apart, for 1 hour RUPTURED MEMBRANES: A gush or trickle of fluid from the vagina should be evaluated, whether or not you have contractions, to determine if your "water has broken". BLEEDING Bright red bleeding that is not mixed with mucus: Normal bloody show is thicker, pink or dark red, and mixed with mucus DECREASED FETAL MOVEMENT: If you notice a decrease in the baby's movement, notify your physician or nurse-midwife or go to the labor unit.

Which of these readings indicates hypertension in the patient whose blood pressure normally is 100 / 60 and MAP of 77? a. 120 / 76; MAP 96 b. 110 / 70; MAP 83 c. 130 / 80; MAP 98 d. 125 / 70; MAP 88

c. 130 / 80; MAP 98 [Rationale: 100 + 30 increase = 130 systolic; 60 + 15 increase = 75, diastolic increased by 20 to 80; an increase of greater than 20 in MAP: 77 + 20 = 97 and this client's map increased to 98.]

Fetal heart rate accelerations indicate ___. (Lecture / powerpoint)

increase in FHR with a return to baseline and indicate fetal well being when 15 bpm for 15 seconds. Accelerations are considered a reassuring acceleration pattern.


Related study sets

Abeka Investigating God's World Chapter 5 Test

View Set

Exam 1 multiple choice questions

View Set

Sonidos en Contexto 20 [f] 21 [t͡ʃ] and 22 [m] [ɱ] [n̪] [n] [ɲ] [ŋ]

View Set

Series 6 Section 1 Debt Securities

View Set

Quizlet for Review Quiz 3 Reconstruction thru the Great Depression

View Set

When Science Fails Literature 108

View Set